You are on page 1of 625
MICROBIOLOGY / PATHOLOGY ‘The Hepatitis A virus, Poliovirus, and Rotavirus have which portal of entry? + Respiratory tract Bled Dsodes Bld Disord Bone Ditters BoDhient Mielaneaut Cats Organs Cats/OrgNeplasms Diseases Dis Onlceviy onic cS Disord Parsee P Fung Pong °Substnce sa “+ Gastrointestinal tract + Skin * Genital tract * Blood eae Disorders| DieDisord "Compounds “Enzymes “Trtains Hemodynamic Dysfinsien Hemody DysSyairones syne toumancogy Immun Tens Tom Infcion Ine Vase Vee Infesion Cont Ic Vins Inestion and Neos nf & Noe Kidney Disses Kid Dis MICROBIOLOGY / PATHOLOGY ir MICROBIOLOGY / PATHOLOGY Vir Which virus causes pneumonia and bronchiolitis in infants? Reoviruses: + Respiratory syncytial virus (RSV) Have 2 unique single-shelled capsid that contains a non-segmented double- stranded DNA genome + Human immunodeficiency virus (HIV) 7 + Have a unique double-shelled capsid that contains a segmented single-stranded RNA genome + Epstein-Barr virus + Varicella-Zoster virus + Have a unique doublo-shelled capsid that contains a segmented double-stranded RNA genome + Have a unique single-shelled capsid that contains a segmented single-stranded DNA genome + Respiratory syncytial virus (RSV) RSV is the only member of the paramyxovirus family (which also includes the ‘measies virus, mumos virus, and perainfluenza viruses) that lacks the envelope glycoproteins hemagglutinin and neuraminidase. Its surface splkes are fusion proteins. Remember: Tho envelope of paremyxoviruses is covered with spike: which contain either hemagglutinin, neuraminidase, or a fusion protein that causes cell fusion and, in some cases, hemolysis. ‘The fusion protein causes cells to fuse, forming multinucleated giant cells (syncytial) ‘which give rise to the name of the virus. Ribavirin (Virazole) is used to treat severely lil hospitalized infants, There Is no vaccine. Parainfluenza viruses cause croup (acute laryngotracheobronchitis) and pneumonia in children and a disease resembling the common cold in adults. The surface spikes: present on the viruses consist of hemagglutinin (H), neuraminidase (N), and fusion (F) proteins. These viruses are transmitted by respiratory droplets and direct contact; there is neither antiviral therapy nor a vaccine available. Important: Interstitial pulmonary inflammation is most characteristic of viral pneumonia. set Cytomegstoviras soncnsctosis syndrome Feeanervs | Yi most nfeena ewe’ tes Maser ise enter asm vue his espraory syst vires [nici el pcomonis in inf Rhino Common ot Vasa vine [crckenp scien [ eps A vine fiepasse a se ison steve in [ies wire Rates smn pps Papen vara) [eentalzocr —snenppiionavinw Palomas roar) Hepa woe icp man inaneceieney vin [AIDS Here pec vin ype? |Harpes genital and neonatal herpes = — lips Troe rere ocr ee [Gyromegalovires [Monomacieoss syndrome or pneumonia + Have a unique double-shelled capsid that contains a segmented double- stranded RNA genome Reoviruses are naked (non-enveloped) viruses with a double-shelled (outer shell and core) icosahedral capsid containing 10 or 11 segments of double-stranded RNA. ‘They replicate in the cytoplasm. ‘There are three groups: 1, Reoviruses —> produce minor respiratory tract Infections and gastrointestinal disease. 2. Rotaviruses (a/so called gastroenteritis virus type B) -> cause infantile diarrhea and are the most common cause of gastroenteritis in children (2 and under). 3. Colorado tick fever virus (arbovirus) > causes Colorado tick fever. Viral gastroenteritis is a self-limiting disease, often referred to as the 24-hour or intestinal flu, An influenza virus does not cause it. It is associated with RNA viruses and is often diagnosed by ELISA on fecal samples. The characteristic symptoms of Viral gastroenteritis inciude sudden gastrointestinal pain, vomiting, and diarthea, Recovery usually occurs within 12:24 hours. Important: Dehydration Is a major pecially in infants, where viral gastroenteritis is sometimes fatal. MICROBIOLOGY / PATHOLOGY Which growth curve describes the lytic reproduction cycle that releases a large number of phage simultaneously? + One-step growth curve + Two-step growth curve + Horizontal growth curve = None of the above crip o20 209" — BOTA. OcoKS MICROBIOLOGY / PATHOLOGY Vir ‘Which two of the following herpes viruses do not cause a vesicular rash, both in primary infections and in reactivations? ‘+ Epstein-Barr virus (EBV) + Varicolla-zoster virus (VZV) ‘+ Herpes simplex virus type 1 (HSV-1) + Herpes simplex virus type 2 (HSV-2) * Cytomegalovirus (CMV) ‘ean ozms- zor — oew. ocx MICROBIOLOGY / PATHOLOGY Vir Which of the following viruses does not cause upper respiratory tract infections, includ- ing symptoms of the common cold? + Paramyxoviruses + Papovaviruses * Coronaviruses + Rhinoviruses + Orthomyxoviruses: MICROBIOLOGY / PATHOLOGY Vir All of the following viruses are paramyxoviruses except: + Mumps virus + Measles virus ‘Influenza A, B, and C vi + Respiratory syncytial virus (RSV) + Parainfluenza virus rt ez08 2007 — pA oCKS + Epstein-Barr virus (EBV) + Cytomegalovirus (CMV) Gingivostomatitis Herpes genitalis Varicella (chickenpox) Infectious mononucleosis [Congenital infection finuero), mononucleosis ++ One-step growth curve The lysis of the bacterial coll releases a large number of phage simultaneously and, consequently the lytic reproduction cycle exhibits a one-step growth curve. The growth curve begins with an eclipse period during which there are no complete ‘infective phage particles. The eclipse period is the time between entry of the vial DNA and formation of the first complete virus within the host cel. The eclipse period is part of a longer period, the latent period, which, like the eclipse period, Begins when the phage injects DNA into a host cell, but which does not end until the first assembled virus from the Infected cell appears extracellularly. ‘The common stages of viral replication include: 1) Attachment 2) Penetration 3) Synthesis 4) Assembly 5) Maturation 6) Release + Influenza A,B, and C viruses ‘The influenza viuses are classified as orthomyxoviruses. Both paramyxoviuses and orthomyx Coviruses are enveloped, single-stranded, negative-stranded RNA viruses. The paramyxoviruses differ from orthomyxoviruses In thet their genomes sre net segmented, they have a larger ‘diameter, and thor surface spikes are different, Note: The cytopathic effect for paramyxoviruses 's syncytia formation (they induce cols to form multinuctoatod giant col). Tata Avis Praia vas Resist ya vn Raker Maine ans ins Rawr nae DNA vir eps plex inp pear vins Vesela int ‘erie + Papovaviruses This family (papovaviruses) contains 2 genera of oncogenic DNA viruses: papilomaviruses and polyomaviruses wich produce latent and chronic infections in their hosts and can Induce tumors in some animal species. ‘The common cold is most frequently caused by thinoviruses (there are over 100 diferent sorooypes: tis is why development ofa vacin so ce) sale caused by cron Mumps is caused by an RNA paramyxovirus and is transmitted via respiratory droplets. It ‘occurs worldwide, with a peak incidence in the winter. The most noticeable symptom of mumps is the painful swelling ofthe paratid glands, either unilatral or biter. Its typically benign and resolves within a woek. Note: Two complicalions are of significance -> orchitis with palnful ‘swelling ofthe testicles in postpubertal males, which can resut in sterlty, and deafness in chi- ren, Influenza is caused by influenza viruses A, B, and C (orthomyxoviruses).. Complications include Reye's syndrome (8 virus) in children [Measles (Ruboo\a) is caused by rubeola virus (RNA paramyxovirus). Characterized by skin ash with Koplik's spots. Transmitted by respiratory droplets. German measles (Rubella) is caused by rubella ius (RNA virus). Transmitted by respiratory droplets. Pharyngitis is an inflammation ofthe pharynx. The main symptom is @ sore throat. Itis caused by a variety of viruses (adenoviruses and coxsackievirises). MICROBIOLOGY / PATHOLOGY MICROBIOLOGY / PATHOLOGY Vir All of the following statements concerning viruses are true except: Influenza virus A has all of the following features except: + A viral nucleic acid (genome) is composed of either DNA or RNA (but not both) + Ahelical capsid that is encased in a protein coat called a capsid + Negative-stranded RNA * They are either naked or enveloped, depending on whether the capsid is surrounded by a lipid bilayer known as an envelope + A lipid envelope of cellular origin + They replicate only in tiving cells and therefore are obligate intracellular parasites + Anonsegmented RNA genome ‘+ Hemagglutinin and neuraminidase spikes embedded on their surfaces + They are sensitive to antibiotics + They depend on host cells for energy production + They cannot be observed with a light microscope + They pass through filters that retain bacteria nt e820) — BATA CAS MICROBIOLOGY / PATHOLOGY MICROBIOLOGY / PATHOLOGY Vir Vir All of the following statements are true except: Which virus can cause herpes zoster lesions along sensory nerve roots in later life? + DNA viruses, with one exception (poxviruses), replicate in the nucleus and use + Smallpox virus the host cell DNA-dependent RNA polymerase to synthesize their mRNA. + Epstein-Barr virus + The genome of all DNA viruses consists of double-stranded DNA, except for the parvoviruses, which have a single-stranded DNA genome + Varicella-Zoster virus + Most RNA viruses undergo their entire replicative cycle in the cytoplasm (except retroviruses and influenza viruses, which replicate in the nucleus of the host coll) “+ Negative polarity is defined as an RNA with the same base sequence as the mRNA eorex 200" OBA. 9S + Negative polarity Is defined as an RNA with the samo base sequence as the mRNA "This is false; positive polarity is defined as an RNA with the same base sequence as the mRNA. RNA with negative polarity has a base sequence that is complimentary to the mRNA. For example, if the mRNA sequence is G-U-C-A, an RNA with negative polar- ity would be C-A-G-U and an RNA with positive polarity would be G-U-C-A. Note: This term positive or negative polarty only refers to RNA viruses, not DNA viruses. Examples of negative polarity RNA viruses include orthomyxoviruses and paramyx- viruses. Important points to remember: The first step in viral gene expression is mRNA synthesis. The positive polarity RNA viruses can use their RNA genome directly as mRNA. The negative polarity RNA viruses must transcribe their own mRNA by using the negative strand as a template. Because the cell does not have an RNA polymerase. capable of using RNA as a template, the virus carries its own RNA dependent RNA polymerase. Note: RNA viruses have a genome which may be single-stranded or double-strande segmented, or nonsegmented. * They are sensitive to antibiotics “This is false; viruses are not sensitive to antibiotics. However, they are sensitive to Interferon, which inhibits their replication. ‘The replication of a virus within a host cell depends on the ability ofthe viral genome to enter ‘the host cell, to remain functional, and to direct the host cell to produce viral macromolecules. Within the host cell, the viral genome achieves control ofthe cel's meta~ bolic activities. The virus then uses the metabolic capacity of the host cell for ‘production of new viruses. Often the replication of a virus causes changes in the host cell, usually causing the death of that cell. Note: For viruses, the burst size is the average number of progeny viruses released per infected cell, All viruses have a central genetic nucleic acid core (either RNA or DINA) surrounded by a protein coat. The genome of the virus may consist of Couble-stranded DNA, single-stranded DNA, single-stranded RNA, or double-stranded RNA. ‘The following are useful in identifying viruses: * Whether or not specitic antisera neutralize the virus (cell culture techniques). Ths is the ‘most generally accoptd aberatory method forthe diagnosis of most commen vial infec: ns. * Morphology of the viral protein coat + Nature of the viral nucieic acid (RNA or DNA) + The abilly of ether or chloroform to inactivate the virus + Varieella-Zoster virus (VZV) Its a member of the herpes virus group. It causes the disease chickenpox (varicella) and shingles (herpes zoster). The virus is vary contagious and may be spread by direct contact or droplets. 90% of all cases of chickenpox occur in children under 9 years of age. Local lesions. (vesicles) occur in the skin after dissemination of the virus through the body. These lesions become encrusted and falloff in about one week. Shingles (herpes zoster) Is the result of reactivation of a latent varicella-zoster virus that may have remained within the body from a childhood case of chickenpox. The Vieus reaches the sen- sory gangiia of the spinal or cranial nerves (most frequently the trigeminal nerve), producing an inflammatory response. It's characterized by painful vesicles that occur on the skin or mucosal ‘surfaces along the distribution of a sensory nerve. It is unilateral, usually involves 1-3 der- matomes, and is more common in individuals who are immunocompromised. Note: Adenosine arabinoside (vidarabine) suppresses the synthesis of varicella-zoster and. herpes simplex viruses, and it tends to diminish new lesion formation and the duration of fever ‘and to prevent the spread of virus through the viscera. Poxvirusos aro ONA viruses that are the largost and most complex animal viruses. This group includes the variola viruses that cause smallpox. Smallpox is an acute, high infectious, often fatal disease that is characterized by high fever, prostralion, and a vesicular, pustular rash, Fortunately, man is the only reservoir for the virus and vaccination with vaccinia, a related poxvirus, has been effective in eradicating smallpox. + Anonsegmented RNA genome Influenza A viruses are the only members of the orthomyxovirus family. The influenza virus is composed of a unique segmented, single-stranded, negative-stranded RNA. ‘genome, a helical nucleocapsid, and an outer lipoprotein envelope. The envelope is Covered with two different types of spikes (which are glycoproteinaceous projections or ‘peplomers), that contain either hemagglutinin or neuraminidase. Influenza viruses are Classified as type A, B, or C, depending on a nucleocapsid antigen. Tho ability of the influenza A virus to cause epidemics is dependent on antigenic ‘changes in the hemagglutinin and neuraminidase. There are two types of changes: antl- genic shifts, which are major changes based on reassortment of genome pieces, and antigenic drifts, which are minor changes based on mutations. Remember: One serious complication associated with outbreaks of influenza is the development of Reye's syndrome, which is an acute pathological condition affecting the central nervous system. It seems to be associated with outbreaks of influenza B virus for unknown reasons. This syndrome is principally associated with children who have taken aspirin to treat the viral infection (again, there is no known direct cause-and-effect relationship between aspirin use and Reye's syndrome). MICROBIOLOGY / PATHOLOGY i MICROBIOLOGY / PATHOLOGY Which of the following is the cause of infectious mononucleosis? Allof the following statements conceming coxsackieviruses are true exes + Streptococci + They belong to the Picornavirus family + Epstein-Barr virus “= They are divided into two groups (A and 8) on the basis of the lesions observed + Rubella viruses. + Group A viruses cause herpangina and hand-foot-and-mouth disease, whereas group B cause pleurodynia, myocarditis, and pericarditis + Paramyxovirus * Group B viruses have a predilection for skin and mucous membranes, whereas group A cause disease in various organs such as the heart, pleura, pancreas, and liver * Their replication is similar to that of the poliovirus oi ez08 2097 BBTAL 9EORS MICROBIOLOGY / PATHOLOGY Vir ‘After the initial primary attack during the early childhood period, the herpes simplex virus remains inactive for periods of time and most commonly resides in the: MICROBIOLOGY / PATHOLOGY Vir Acquired immunodeficiency syndrome (AIDS) is caused by an enveloped, single-stranded, linear, positive-polarity RNA virus known as’ + An arbovirus + Submangibular ganglion + Trigeminal ganglion + Aparvovirus +A retrovirus + Sublingual ganglion * Ciliary ganglion + An adenovirus capram e227 — pan. ceoxs eon es08-t00r— pera. BeOS ‘Trigeminal ganglion The inactive virus resides in sensory nerve ganglia (most commonly, the trigeminal ganglion), but will often reappear later as the familiar “cold sore”, usually on the outside ofthe lips. This disease is referred to as “recurrent herpes labialis”. Emotional stress, trauma, and excessive exposure to sunlight have been implicated as factors for the appearance of the recurrent herpetic lesions on the lip. Acyclovir 5% ointment (Zovirax) has been successful in reducing the duration and severity of these sores. Remember: 1. Horpes Simplex Type | (primary herpetic gingivostomatitis, recurrent herpes labialis) Is transmitted by direct contact. It affects the lips, face, skin, and oral mucosa (above the waist). HSV-I becomes latent in the trigeminal ganglion. 2. Herpes Simplex Type Il (herpes genitals) is spread by sexual contact. It affects the mucosa of the genital and anal regions (below the waist). HSV-2 becomes latent in the lumbar and sacral ganglia. 3. A tzanck smear is a diagnostic test that reveals intranuclear inclusions in herpes virus infections. Genital herpes may have serious consequences in pregnant women because the virus ‘can be transmitted to the infant during vaginal delivery. The virus can cause damage to the infant's central nervous system and / or eyes. + Epstein-Barr virus. The Epstein-Barr virus (EBV) is a member of the herpes virus group. It causes infectious mononucleosis and has been associated with the subsequent development Of two forms of cancer: Burkitt's lymphoma and nasopharyngeal carcinoma. EBV Is also associated with hairy leukoplakia, a whitish, nonmalignant lesion on the tongue seen especially in AIDS patients. The virus specifically infects B-lymphocytes ‘and some epithelial cells. It is associated with the production of atypical lymphocytes. ‘and IgM heterophile antibodies identified by the heterophile test (also called the ‘mononucleosis spot test). This antibody eventually appears in the serum of more than 80% of the patients with infectious mononucleosis, hence itis highly diagnostic of the disease. Remember: 1, Rubella viruses cause German measles (rubella), which present with a characteristic rash (flat, pink spots on the face and then spreads to other body parts) 2. Paramyxoviruses can cause measles (rubeola) and mumps, Rubeola is character- ized by the formation of Koplik’s spots in the oral cavity. These spots are small, bluish-white lesions surrounded by a red ring. They cannot be wiped off and occur opposite the molars. Mumps cause enlargement of the parotid glands, Serious com- plications include deafness in children and orchitis (inflammation of the testis) in males past puberty. “+A retrovirus Retroviruses are RNA viruses that have their genome surrounded by an inner proteln envelope and an ‘Cuter envelope that contains lipid and alycoprotan spikes, which serve to attach te viru othe host col, ‘The word reo" refers tothe possession ofthe enzyme reverse transcriptase (an RNA-drecied DNA ‘polymerase, wich ranseribas RNA to DNA during the procoss of viral nucoic acid eynthecis. ‘There ae three groups: 41.Oncovirus group (0.9. HTLV) > produce leukemia, Iymphomas, breast carcinomas, and sarcomas. 2 Lentivirus group (0g, IVA and HIV-2) -» causes AIDS. 43. Spumavirus group there are no known human pathogens. “The transmission of HIV occurs primarly by sexual contact and by transfer of infected blood. The virus infects and kil helpor (CD4) T-cells, rosuttng inthe depression of both humoral and cell modated in ry. travels throughout te body, particulary in macrophages. It induces a distinctive CPE (cytoparhic effect) called glant-cell (syncytia) formation. Important: The rapid emargance of new strains of HIV is primary the resi of frequent errors inro= duced by viral reverse transeriptase. Notes: 1 The env gone of retrovisuses codes for the retroviral envelope proteins, the pol gene codes reverse transcriptase and the gag gone codes fra protein thats the precursor to four stuctral p tains ofthe nucsooid, 2. Acute HIV-1 primary infection can present as a mononucleosissllke syndrome with fever, fatigue oro that, and skin rash, 3. HIV cites from the RNA tumor vuses in that it yses the host calls. RNA tumor viruses transform th} ‘cals that they invade but do not possess cytolytic activity. predilection for skin and mucous membran ‘the heart, pleura, pane + Group B viruses have ‘A-cause disease in various organs such , whereas group ‘and liver ‘This s false; group A viruses have a predilection for skin and mucous membranes, whereas ‘group B cause disease in verious organs such as the heart, pleura, pancreas, and liver. ‘The picomaviuses are very smal, non-enveloped, and are composed of a positive stranded, single stranded RNA genome. This family incudes the enteroviruses (poliovirus, coxsackievituses, ‘echoviruses, and the hopattisAvitus) and the rhinovruses. Note: The picomavruses are incapable Cf causing cell transformation, due to the RNA genome. Coxsackloviruses: + Group A> cause herpangina and haned-footand-mouth disease. Note: The locaton of oral lesions Aistinguishes these two diseases from one another the oral lesions of herpangina appear on the throat, palate, or tongue; the oral lesions of hand-foot-and mouth disease appear on the buccal ‘mucosa ane gingiva. + Group B cause pleurodynia, myocarditis, pericardts, and juvenie diabetes. Echoviruses cause aseptic (viral) meningitis, upper respiratory infections, and severe diarhea in rnewoors. ‘The poliovirus causes poliomyelitis, while the Hepatitis A virus causes infectious hepatts. Rhinoviruses are the main cause ofthe common cold. There are more than 100 serologic types, which is the main reason that vaccine has not been found. MICROBIOLOGY / PATHOLOGY Vir MICROBIOLOGY / PATHOLOGY Vir Which of the following DNA enveloped viruses are the largest and most complex? A highly Infectious viral disease that chiefly affects chilren and leads to paralysis, ‘muscular atrophy, and often deformity is called: + Herpes viruses = Mumps: + Hepatitis B virus + Measles + Poxviruses + Poliomyelitis + Infectious mononucleosis, ‘xo ez08- 00" — BATA. DECKS rom 2208 2207 — oN. BeOS MICROBIOLOGY / PATHOLOGY MICROBIOLOGY / PATHOLOGY Vir ‘The cytopathic effect that is seen when a virus infects a specific cell culture: Vir Allof the following diseases are associated with adenoviruses except: ‘+ Acute respiratory infections + Is the same for most viruses. + Is not useful in diagnostic virology + Acute contagious conjunctivitis (pinkeye) + Generalized systemic disease with a maculopapular rash “+ Is characteristic of each virus and can be used for detection of that virus + Pharyngoconjunctval fever cheracterized by fever, pharyngitis, and conjunctivitis + Does not affect the specific infected cell ‘cori ett 2007 BOTA, DECKS CCopyraht ©2004 — DENTAL DECKS + Is characteristic of each virus and can be used for detection of that virus ‘The hallmark of viral infection of the cell is the cytopathic effect (CPE). This change starts with alterations of cell morphology accompanied by marked derangement of cel function and culminates in the lysis and death of cells. Important: Not all viruses cause CPE; some can replicate while causing title morphologic or functional change in the cell ‘Such changes include: + Necrosis + Detachment from substrate + Hypertrophy + Lysis * Giant cell formation + Membrane fusion + Hypoplasia + Altered membrane permeability + Metapiasia + Inclusion bodies + Altered shape + Apoptosis ‘These cytologic changes provide useful presumptive evidence for the diagnosis of the viruses that induce the cytopathic effects Important: Viruses use specific cell surface receptors to bind to and subsequently gain entry into their host cells. The identification of these receptors explains the cellular tropism of viruses. + Poliomyelitis (Polio) Itis caused by the poliovirus, which is an enterovirus that affects the anterior horn cells (motor cells) of the spinal cord. This virus has an affinity for motor neurons. It is transmitted by consumption of water with fecal contaminants. It replicates in the mucosa Of the pharynx and Gl tract before entering the blood. It then travels to the CNS and infects the anterior horn cells. It is uncommon in the U.S. due to successful vaccination programs initiated in the 1950s" throughout the world. The initial symptoms of poliomyslitis include headache, vomiting, constipation, and sore throat. Paralysis may follow and is asymmetric and flaccid. Note: Two vaccines are currently used for active immunization against poliomyelitis, the inactivated poliovirus vaccine (/PV) and a trivalent live oral poliovirus vaccine (OPY). Both vaccines produce immunity in more than 90% of the recipients. + Goneralized systemic disease with a maculopapular rash, ‘Adenoviruses are naked (non-enveloped) medium-sized viruses composed of an icosahedral ‘nucleocapsid and a double-stranded linear DNA genome. They have spikes (glycoprotein- ‘aceous projections that inthis case are hemagglutinin proteins) protruding from thelr surfaces ‘that are Involved in the absorption or attachment of the virus to the host call. These viruses ‘requently cause subclinical infections. These viruses are transmitted by aerosol droplet, the fecal-oral route, and direct inoculation. They are classified into seven groups (A through @). ‘Adenoviruses are commonly associated with particular ciical syndromes: Disease AL Risk Acute Respiratory Illness Miltary recruits, boarding schools, etc. Pharyngitis, Infants Gastroenteritis Infants Conjunctivitis al Pneumonia Infants, miltary recruts Keratoconjunctivitis al ‘Acute Hemorrhagic Cystitis Infants Hepatitis Infants, liver transplant patients Notes: }1.The ability of pathogenic microorganisms, Including viruses, to attach to and invade} cols and tissues establishes specific tissue affinities for pathogenic 2. Most viral antigens of diagnostic value are proteins. ‘The poxvirus family includes three viruses of medical importance: smallpox virus (variola virus), vaccinia virus, and molluscum contagious virus (MCV). Poxviruses are the largest and most complex viruses. Poxviruses are very large (400-nm X 230-nm), brick-shaped particles containing an ‘enveloped linear double-stranded DNA genome. These viruses multiply in the cytoplasm of host cells (as opposed to the nucleus) and are usually associated with skin rashes. The most important human poxvirus is smallpox (variola). Smallpox is an acute, highly infectious, ‘often fatal disease that fa characterized by high fever, proatration, and a vesicular, pustular rash. This disease has been eradicated by global use of the vaccine, which contains live, attenuated vaccinia virus. 1. The virion of poxviruses contains a DNA-dependent RNA polymerase. This enzyme required because the vitus replicates in the cytoplasm and does not have access to the cellular RNA polymerase, which is located in the nucleus. ‘Smallpox virus has @ single, stable serotype, which isthe key to the success of th vaccine. If the antigenicity varied as it does in infuenza virus, eradication would not} have succeeded, Herpesviruses, adenoviruses, and papovaviruses are spherical or ovoid viruses and] ‘multiply in the nucteus of host cals. MICROBIOLOGY / PATHOLOGY Herpes viruses can be described as: MICROBIOLOGY / PATHOLOGY ‘The infectious viral particle is called a: Vir + Large-sized enveloped viruses with an icosahedral nucleocapsid containing = Viroid circular, single-stranded DNA * Virion ‘+ Medium-sized enveloped viruses with an icosahedral nucleocapsid containing linear, double-stranded RNA * Prion ‘+ Small-sized enveloped viruses with an icosahedral nucleocapsid containing + Noon circular, double-stranded DNA ‘+ Medium-sized enveloped viruses with an icosahedral nucleocapsid containing linear, double-stranded DNA eon ez 2007 — BOTA DECKS eprintezee 2007 pea. cecKs MICROBIOLOGY / PATHOLOGY Vir The leading cause of childhood gingivostomatitis in children ages 1 to 3 is: MICROBIOLOGY / PATHOLOGY Vir All of the following statements concerning herpes simplex type 1 are true except: + Many children have asymptomatic primary infections + Herpes simplex + May be diagnosed by a Tzanck smear for rapid identification when skin lesions. * Chickenpox are involved + Influenza + May involve a primary infection (e.9.. gingivostomatitis) or a recurrent infection (e.g. cold sores) + Rubella + Can be treated prophylactically by a vaccine enn szns- 2097 ETAL DEERE lt e208- 207 — Dena OS + Can be treated prophylactically by a vaccine ~T false; it cannot be treated prophylactically by vaccine. The inactive HSV-virus resides in sensory nerve ganglia (most commonly, the trigeminal ganglion), but will often reappear later as the familiar “cold sore,” usually at the mucocutaneous |unction of the lips This disease is referred to as “recurrent herpes labialis.” Emotional stress, trauma, and excessive exposure to sunlight have been implicated as factors for the appearance of the recurrent herpetic lesions on the: lip. Acyclovir 5% ointment (Zovirax) has been successful in reducing the duration and. severity of these sores. HSV-1 can also cause the following recurrent infections: keratoconjunetivitis. and ‘encephalitis. Remember: The primary infection can range from subclinical (asymptomatic) to severe systemic infections. Genital herpes (caused by HSV-2) may have serious consequences in pregnant ‘women because the virus can be transmitted to the infant during vaginal delivery. The virus can cause damage to the infant's central nervous system and / or eyes. ‘+ Medium-sized enveloped viruses with an icosahedral nucleocapsid containing linear, double-stranded DNA The Herpes Virus family contains five Important human pathogens: herpes simplex virus types 1 and 2, varicella-zoster virus (VZV), cytomegalovirus (CMV), and Epstein-Barr virus (EBV). All herpesviruses are morphologically identical. They have a large double stranded DNA genome. The virion consists of an icosahedral ‘nucleocapsid of about 100 nm in diameter, which is surrounded by a lipid bilayer envelope. Between the capsid and the envelope is an amorphous layer of proteins, termed the tegument. They replicate in the nucleus of the host cell and are the only ‘viruses to obtain their envelopes by budding from the nuclear membrane. Its a characteristic of all herpesviruses that, following primary infection, the virus establishes a latent infection in the host and may reactivate at any stagé Reactivation is frequently, but not always, associated with further disease. ‘Three of the herpes viruses, herpes simplex 1 and 2 and the varicella-zoster virus, cause a vesicular rash. Certain herpes viruses have oncogenic potential (causing cancer) in humans. The Epstein-Barr virus is associated with Burkitt's lymphoma end nasopharyngeal carcinoma, ‘+ Herpes simplex The primary infection (primary herpetic gingivostomatits) usually occurs in a child under ten years of age who has had no contact with the Type | herpes simplex virus ‘and who therefore has no neutralizing antibodies. it may also affect young adults (15- 25). Nearly all primary infections are of the subclinical type (they may only have flu- ‘tke symptoms) and one or two mild sores in the mouth which go unnoticed by the parents, In other children, the primary infection may be manifested by acute symptoms (acute herpetic. gingivostomatitis). These include fever; irritability; cervical Iymph- adenopathy; fiery red gingival tissues; and small, yellowish vesicles which rupture and result in painful ulcers on the free and attached mucosa. The most serious potential problem in a child with this infection is dehydration due to the child not ‘wanting to eat or drink because of the pain, ‘Treatment is supportive and aimed toward the relief of the acute symptoms so that fluid and nutritional intake can be maintained. Primary herpetic gingivostomatitis usually runs a course of 14-20 days and the ulcers heal without scarring. Remember: Of all of the herpes viruses, herpes simplex viruses 1 and 2 cause manifestations of recurrent infection in otherwise healthy people. *Virion {In contrast to bacteria, fungi, and parasites, viruses are not cells, ic., they are not capable of reproducing independently, do not have a nucleus, and do not have ‘organelles such as ribosomes, mitochondria, and lysosomes. Viruses are smaller than cells and cannot be seen in the light microscope. Note: Almost all viruses are haploid (contain a single copy of their genome, the exception Is the retrovirus family, whose members are diploid). ‘The viral particles, or virions, contain either DNA or RNA that is encased in a protein coat called a capsid. They are either naked or enveloped, depending on whether the capsid is surrounded by a lipoprotein envelope. The capsid is composed of polypeptide units called capsomeres. Some viruses (orthomyxoviruses and paramyxoviruses) have envelopes that are covered with spikes, which contain either hemagglutinin, neuraminidase, or a fusion protein that causes cell fusion and, in some cases, hemolysis. Viroids consist solely of a single molecule of circular RNA without a protein coat or envelope. They cause several plant diseases but are not implicated in any human disease. Prions are infectious protein particles that are composed solely of protein. They ‘cause certain “slow” diseases such as Creutzfeldt-Jakob disease in humans and scrapie in sheep. MICROBIOLOGY / PATHOLOGY MICROBIOLOGY / PATHOLOGY A bacteriophage with the ability to form a stable, nondisruptive relationship within a bacterium is called a: Vir A bacteriophage: + Is a bacterium that phagocytizes other organisms + Virulent phage + Is a bacterium that becomes phagocytosed by other organisms + Plasmid + Is a virus that infects bacteria + Temperate phage + Is a fragment of DNA + Phage T4 cr eee -207 — DENTAL DKS crepe ons- 27 — OO eos MICROBIOLOGY / PATHOLOGY MICROBIOLOGY / PATHOLOGY Vir Which group of viruses possess an RNA genome that does not function as a pos- itive or negative sense molecule but acts as a template for the production of viral DNA? The usual site of latency for the Herpes simplex virus type 2 Is: + The cranial sensory ganglia (trigeminal) * The lumbar or sacral sensory ganglia ecoronarinses + The cranial or thoracie sensory ganglia + Picomaviruses + Buymphocytes + Retroviruses + Togaviruses apr e20e- 207 — DaH. Des rr 0238-2087 — DENTAL OS * Temperate phage Itis a bacteriophage which is capable of replication by an alternate method in which the phage genome is incorporated into the bacterial chromosome. It persists through many cell divisions of the bacterium without destroying the host, in contrast to a virulent phage that lyses and kills its host. Remember what a bacteriophage is -> a virus that can replicate only within specific host bacterial cells. lysogenic bacterium is one that contains a temperate bacteriophage. (Example is Corynebacterium diphtheriae, see below.) Lysogenic conversion —> the alteration of a bacterium to a virulent strain by the transfer of a DNA temperate bacteriophage. The classic example of this conversion is in the alteration of Corynebacterium diphtheriae to a virulent strain. The presence of this temperate phage renders the bacterium pathogenic (without the phage it would not be harmtu) 1. Enterobacteria phage T4 is a phage that infects E. coli bacteria. 12. Plasmids are extrachromosomal, double-stranded, circular DNA molecules thal are capable of replicating independently of the bacterial chromosome, + The lumbar or sacral sensory ganglia cranial sensory ganglia irigemina) | Via respiratory secretions and saliva Lumbar orsscrl sensory ganglia | Sexual contact, perinatal infetion [Cranial or thoracic sensory ganglia _| Via respiratory sections a iymphocyten Vi esprtory serene and alive certain intrauterine infection ransfsions.sex- ual contact, via seretions (¢g, salva land wri) + Retroviruses "This group includes the Human immunodeficiency virus (HIV). ‘This is achieved by an RNA-dependent DNA polymerase (reverse transcriptase) that is packaged with the RNA genome. The resulting viral DNA integrates into the host cell genome to provide the template for viral RNA synthesis by hostderived mechanisms. Note: In order for RNA oncogenic viruses to be intregrated into the host genome, they must possess an RNA-dependent DNA polymerase (reverse transcriptase). Important points to remember: + For RNA virus Transcription occurs in the cytoplasm except for retroviruses and influenza =Transcription involves an RNA-dependent RNA polymerase oxcopt for retroviruses, which as explained above possess the enzyme reverse transcriptase (an RNA-dependent DNA polymeraso). + For DNA viruses: = Transcription occurs in the nucleus except for poxviruses. = Transcription involves a host-cell DNA-dependent RNA polymerase. ‘Is a virus that infects bacteria ‘A bacteriophage is a very delicate bacterial virus which may attack and destroy bacte- ‘ial cells under certain conditions. It contains a nucleic acid core (DNA or RNA) and a protein coat. Some have a taiblke structure through which they inject their nucleic acid into the bacterial host cell. Its also called a phage. ‘A host coll is a cell within which a virus replicates. Once inside the host cell, the viral genome achieves control of the cell's metabolic activities. The virus then uses the ‘metabolic capacity of the host cell to reproduce new viruses. Often the replication of these new viruses causes death of the host cell. Bacteriophage infection follows one of two courses —Iysis or lysogeny. Ifthe infecting Virus muttipies within the host cell and destroys it, the virus is said to be lytic, or virulent. On the other hand, if the virus does not replicate but rather integrates into the bacterial chromosome, the virus is said to be temperate, or lysogenic. The phage in the lysogenic cycle can spontaneously become lytic. The presence of the integrated virus, which is called a prophage, generally renders the cell resistant to infection by similar phages. Lysogeny does not result in the destruction of the host cell. Remember: Transduction Is the transfer of DNA — from a donor cell to a recipient cell —with the DNA packaged within a bacteriophage. MICROBIOLOGY / PATHOLOGY Terms ‘An increase in the size of an organ or tissue due to an increase in the size of cells is known as: + Hypertrophy = Atrophy + Hyperplasia “+ Aplasia + Hypoplasia + Metaplasia unig 9208-27 — ETAL BeOS MICROBIOLOGY / PATHOLOGY ‘An interactive association between two populations of different species living together in which one population benefits from the association, while the other is not affected is called: Terms + Symbiosis + Mutualism * Commensalism * Latency MICROBIOLOGY / PATHOLOGY Vir All of the following are DNA enveloped viruses exce} + Herpes viruses + Human immunodeficiency virus (HIV) + Poxviruses: + Hepatitis B virus MICROBIOLOGY / PATHOLOGY Missense mutations: Terms + Stop protein synthesis prematurely + Result in the substitution of one amino acid for another + Generate a termination codon + Almost always destroy protein function roi e206. por oar oeces + Commensalism ‘A.common example would be the removal of oxygen from a habitat, as a result of the ‘metabolic activities of a population of facultative anaerobic populations. This creates an environment favorable for the growth of obligately anaerobic populations. ‘Symbiosis > an obligatory, interactive association between members of two popula tions, producing a stable condition in which the two organisms live together in close physical proximity. it may, but does not necessarily, beneft each member. ‘Mutualism —> is a form of symbiosis, whereby both members live together with mutual benefit. Latency —>is a slate of dormancy; existing as a potential, as in tuberculosis or herpes. ‘simplex viral infection. They may be latent for extended periods of time and become. active under certain conditions. + Hypertrophy Remember these term: Atrophy is a decrease in the size of an organ or tissue resulting from a decrease in mass of preexisting cells. It most offen results from disuse, aging, or a disease process. Hyperplasia is an increase In the size of an organ or tissue due to an increase in the number of cells. Aplasia is a failure of cell production. During fetal development, aplasia results in agenesis (the absence of an organ). Hypoplasia is 2 decrease in cell production less extreme than aplasia. Metaplasia is the process whereby one cell type changes to another cell type in response to stress and generally assists the host to adapt to the stress. The most ‘common type of epithelial metaplasia involves replacement of columnar cells by Stratified squamous epithelium. + Result in the substitution of one amino acid for another In the living cell, DNA undergoes frequent chemical change, especially when it is being replicated (in S phase of the eukaryotic cell cycle). Most of these changes are quickly repaired. Those that are nct result in a mutation. Thus, mutation is a failure of DNA repair. Mutations may be caused by various mutagens, including UV light, radiation, chemicals, ‘and certain viruses. Mutation types include base substitution (nucleotide substitutions), frame shift mutations, insertions (transposons), and deletions. Base substitutions result in two types of mutations: 4. Missense mutation -> occurs when the base substitution results in a codon that simply causes a different amino acid to be inserted 2. Nonsense mutation -> occurs when the base substitution generates a termination codon that stops protein synthesis prematurely. These mutations almost always destroy protein function, involving base substitution in which the orientation of purine and pyrimidine is reversed (a purine is replaced by al Dyrimicine ora pyrimidine by ® purine) {transition mutation is @ point mulation involving substitution of one base pai fq fanather by replacement of one purine by another purine and of one pytimisine by} another pyrimidine but without change inthe purne-pyrimidine oxentation. + Human immunodeficiency virus (HIV) RNA enveloped viruses: + Respiratory viruses -> influenza A and B, parainfuenza viruses, respiratory syncytial virus 3, mumps, rubella, and hepatitis C viruses + Rabies virus and HIV (numen immunoderfeiency vis) ‘The vius family Picornaviridae includes the folowing RNA non + Enteroviruses ~ poliovirus, coxsackieviruses, and echoviruses. + Rhinoviruses -» cause the common cole + Reoviruses ~ rotavirus, causes viral gastroenteritis in young chidron * Hepatitis A -> causes infectious hepatitis nveloped viruses: DNA enveloped viruses: ‘Herpes viruses (Herpes simplex, Varicella-zoste, Epstein-Barr, and Cytomegalovirus) + Poxviruses (Smallpox and Vaccinia virus) + Hepatitis B virus DNA non-enveloped viruses: + Adenoviruses ~> cause upper and lower respiratory infections “+ Papillomaviruses (papovaviruses) -> cause pepilomas (warts) on the skin and mucous ‘membranes. Human papilomaviruses 16 and 18 are strongly correlated withthe appearance of ‘cervical cancer. + Parvovirusos -> cause erythema infectiosum (slapped-chseks syndrome, fith disease), aplastic crisis, and fetal infections. MICROBIOLOGY / PATHOLOGY Bact MICROBIOLOGY / PATHOLOGY Which bacteria produce a toxin that can be detected using the ELISA assay? {Al of the following are important features of bacterial spores except: + Bacteroides += They are highly resistant to heating + Eschetichia col + They are highly resistant to many disinfectants. “+ Neisseria + They are produced only by members of two genera of bacteria of medical importance, Bacillus and Clostridium, both of which are gram-positive rods “+ Ekenella * They can survive for many years, especially in soll * They exhibit a great amount of metabolic activity “= They contain dipicolinie acid (calcium aipicolinate), a calcium chelator that is found Virtually nowhere else in the biological world eprom 2007 BOWL DES alo e208 2007 ORAL DECKS MICROBIOLOGY / PATHOLOGY Bact ‘The process in which DNA released by lysis of one bacterium is taken up by @ second bacterium, leading to a change in phenotype ofthat second bacterium, is called MICROBIOLOGY / PATHOLOGY Bact ‘Which ofthe following is the region in prokaryotes where the DNA is concentrated? + Granules ‘Transduction = Nucleoid “+ Conjugation + Plasmids “Transformation += Transposons * Sexduction ‘ono cate-2007— Daa. sens ‘ror an n0— AL.DECRS ‘= Transformation ‘Transformation is the most primitive of mechanisms for gene transfer among bacteria. It ‘occurs naturally among some bacteria. Its used in the laboratory to create recombinant DNA and is also used to study the effects of introducing DNA into a call, and in mapping ‘gene locations. There is no requirement for cell-to-cell contact. Transformed cells are ‘cols that have incorporated DNA by transformation. Example: when rough pneumococci ‘grown in the presence of DNA from dead smooth pneumococci develop capsules. Note: The DNA that is picked up the recipient cell must be double-stranded. As it enters the cell, an intracellular DNAase (endonuclease) degrades one of the strands. This hydrolysis provides the energy to pull the rest of DNA into the coll. Uptake is dependent upon the presence of a protein known as competence factor. Once inside the cell, the now single-stranded DNA can insert into homologous regions of the recipient's chromosome, Remember: There are three processes for genetic transfer in bacteria: transformation, transduction, and conjugation. All of these are significant in that they bring about an increase in the amount of genetic variation within a population, ‘Notes: }1. Sexduction is genetic transfer mediated by F-prime cells. Transduction is the transfer of genetic material from one bacterial cell o another by| viral infection. + Escherichia coli ELISA (enzyme-linked immunosorbent assay) is a sensitive immunoassay that uses an ‘enzyme linked to an antibody or antigen as a marker for the detection of a specific protein, especially an antigen (toxin). Vibrio cholerae, which is a curved, gram-negative bacilus, produces an enterotoxin that ‘causes cholera (an acute diartheal disease). This enterotoxin can also be detected using the ELISA assay. Staphylococcus aureus produces an enterotoxin that causes an acute-onset food poisoning. This enterotoxin can also be detected using the ELISA assay. Remember: E. coli are short, gram-negative, facultative anaerobic rods and are motile by means of a peritrichous flagella. They are normally present in the intestines of humans. Enterotoxin-producing strains of E. coli are capable of causing mild and severe: forms of enterocolitis. E, coll s the most common cause of urinary tract infections (cystitis). Note: Bacterial toxins are the most potent poisons known and their potency is paral- leled by their efciency as antigens. + Nucleoid Itis a membraneless structure or region in prokaryotic cells that contains DNA but does not itself replicate. The nucleoid contains no nuclear membrane, no nucleolus, no mmtotic apparatus, and no histones, so there is little resemblance to the eukaryotic nucleus. Granules (inclusion bodies): The cytoplasm contains several cifferent granules that serve as storage arcas for nutrients. For example, volutin (which is also called ‘metachromatic Granules) is a reserve of high eneray stored in the form of polymerized ‘metaphosphate that can be used in the synthesis of ATP. These metachromatic ‘granules are commonly associated with Pseudomonas aeruginosa and Corynebacterium diphtheriae. Other examples of inclusion bodies include sulfur ‘granules and PHB (poly-hydroxybutyric acid), Plasmids are extrachromosomal, double-stranded, circular DNA molecules that are capable of replicating independently of the bacterial chromosome. ‘Transposons are pioces of DNA that move readily from one site to another, either within ‘or between the DNAS of bacteria, plasmids, or bacteriophages. They represent genes. that offen encode proteins necessary for antibiotic resistance. Transposition is a ‘mechanism of inserting a transposon into another molecule of DNA. ‘They exhibit a great amount of metabolic activity ‘This s false; they exhibit no measurable metabolic activity Bacterial spores (or endospores) are the toughest forms of fe known. Only G+ cals form spores, ‘spectically members of tho ganera Bacilus and Clostridium, Spores ar formed by bactora to sur- Vive during periods of deprivation, such asthe loss ofa food or water supply. When a spore-frming ‘bacterium (SFB) senses that tough times are coming a series of complex events are triggered that lead to the formation ofa spore. Basically a spore isa structure that contains the absolute minimum ‘of genstic information and associated materials required to produce the vegetative form once times, ‘become good again. ete 1. Spores are primitive, thick walled, usually uniceular cells by which bacteria, fungi, and green planis reproduce. la By means of a process called asexual reproduction, spores are able to grow into neu! ‘organisms without uniting with another reproductive coll [3 Active spores have thin cel walls: dormant spores have thick, strong cell walls Example: Bacterial endospore is a heat-resistant spore formed within the cal. The endospore is a Complex, multlayered structure containing peptidoglycan within ts complex spore coat and calcium dlicolinate within its core. This bacterial endospore Is very difficult to destroy (more so than the HIV, HBY, and the TB virus). To destroy it you must autoclave at the proper temperature (121°C for 20 minutos), Note: Anthrax is caused by Bacilus anthracis. Botulism, gas gangrene, and tetanus are caused by Clostridium botulinum, C. perringons, and C. ttan, respectively. MICROBIOLOGY / PATHOLOGY ‘The enzymes superoxide dismutase and catalase are present in MICROBIOLOGY / PATHOLOGY Bact ‘Two different pathways are involved in the metabolism of glucose: one anaerobic and ‘one aerobic. The aerobic process occurs in the: + Obligate aerobes only ‘Mitochondria and is very efficient + Facultative anaerobes only ‘= Cytoplasm and is only moderately efficient * Obligate anaerobes only ‘= Mitochondria and is not efficient. * Both obligate aerobes and facultative anaerobes + Cytoplasm and is very efficient cnt ca 207 — ena eos apt ez08-2007— ora ons MICROBIOLOGY / PATHOLOGY MICROBIOLOGY / PATHOLOGY Bact All ofthe following are prokaryotic cells except: Al of the following bacteria have been found to be the principal bacteria associated with ‘adult periodontitis except: * Bacteria + Porphyromonas gingivalis * Mycoplasmas + Prevotelia intermedia + Fungi + Capnocylophaga ochraceus + Rickettsia + Bacteroides forsythus + Chiamydia + Campylobacter rectus + Fungi Cells have evolved into two fundamentally different types, eukaryotic and prokaryotic. The eukaryotic cell has a true nucleus with multiple chromosomes surrounded by a nuclear ‘membrane and uses a mitotic apparatus to encure equal allocation of the chromosomes. to progeny cells. Examples include: human cells, plants, animals, protozoa, and fungi In the prokaryotic cell the nuclear material is not contained within a nucleus and consists of a single circular molecule of loosely organized DNA lacking 2 nuclear membrane and mitotic apparatus. Examples include: bacteria, mycoplasmas, rickettsia, and chlamydia. Important points to remember: * Eukaryotic cells contain organelles, such as mitochondria and lysosomes, and larger (805) ribosomes, whereas prokaryotes contain no organelles and smaller (70S) ribosomes. ‘+ Most prokaryotes (except Mycoplasmas) have a rigid external cell wall that contains peptidoglycan. Eukaryotes do not contain peptidoglycan. + Eukaryotes replicate by mitosis, while prokaryotes replicate by binary fission. +The eukaryotic cell membrana contains sterols, whereas no prokaryote, except Mycoplasmas, has sterols in its membranes. Remember: Viruses are not cells; they are obligate intracellular parasites. They contain either RNA or DNA, do not contain organelles, and have a protein capsid and lipoprotein envelope. + Mitochondria and Is very efficient “The anaerobic cycle takes place in the cytoplasm and is only moderately efficient. Respiration refers to the method of obtaining metabolic energy that involves an ‘oxidative phosphorylation. It involves the formation of ATP during electron transfer. It can be aerobic (molecular oxygen is the terminal hydrogen acceptor) or anaerobic (nitrate or sulfate isthe terminal hydrogen acceptor). Aerobic respiration involves a cell membrane respiratory (electron transport) chain, This chain Is composed of cytochromes, flavoproteins, lipid cofactors, and coupling factors. The electron transport chain is present in the inner mitochondrial membrane and 's the final common pathway by which electrons derived from different fuels of the body flow to oxygen. Fermentation is 2 method by which some bacteria obtain metabolic energy. It is characterized by substrate phosphorylation. It involves the formation of ATP not coupled to electron transfer. It requires an intermediate product of glucose metabolism {often pyruvate) as the final hydrogen acceptor. Key point: In fermentation, the final electron acceptor is an organic compound. + Capnoeytophaga ochraceus (Other bacteria associated with adult periodontitis incude: ‘+ Fusobactorium nucleatum + Spirochetos: ‘Note: Current AAP guidelines recommend replacing “adult” periodoniis with “chronic” periodontitis. PPrinelpal bacteria associated with refractory periodontitis: + Porphyromonas gingivalis formerly known as Bacteroides gingivalis) Bacteroides forsythus + Campylobacter rectus + Prevoela intermedia, ‘Note: Current AAP guidelines recommend the elimination of a separate disease category for “refractory periodontitis’, replacing It wit refractory chronic periodontitis, refractory aggressive periodontitis, ete, Principal bacteria associatod with rapidly progressive periodontitis: * Porphyromonas gingivalis, “= Eubacterium (brachy, nodetum, and timidur) + Provotella intermedi + Fusobacterium nucleatum + Campylobacter rectus + Elkonella corrodens “Rapidly progressing periodontitis is most commonly s8en in young adults (20-35 years ol) and is characterized by marked inflammation, rapid bone loss, and periods of spontancous remission. Most ofthese pationts have depressed neutrophil chemotaxis + Both obligate aerobes and facultative anaerobes ‘Two toxic molecules arise as a byproduct of aerobic metabolism, hydrogen peroxide and free superoxide radicals. Cells possess an elaborate defense system to destroy these toxic molecules, including enzymes such as catalase and superoxide dismutase. ‘Superoxide dismutase catalyzes the decomposition of free superoxide radicals into water and hydrogen peroxide, which is subsequently degraded by catalase. Catalase calalyzes the decomposition of hydrogen peroxide to water and oxygen. Obligate anaerobes cannot grow in the presence of oxygen because they lack either ‘superoxide dismutase or catalase or both. They cannot carry out respiratory metabolism. In the mouth, these microorganisms are considered to be part of the normal microflora and opportunistic. Obligate aerobes require oxygen to grow because their ATP-generating system is dependent on oxygen as the hydrogen acceptor. Facultative anaerobes utlize oxygen to generate energy by respiration if tis present, bbut they can use the fermentation pathway to synthesize ATP in the absence of sufficient oxygen. MICROBIOLOGY / PATHOLOGY Bact ‘Which two bacteria have been found to be the principal bacteria associated with acute necrotizing ulcerative gingivitis (ANUG)? MICROBIOLOGY / PATHOLOGY Bact ‘Which toxin or enzyme produced by Group A beta-hemolytc Streptococci (S pyogenes) activates plasminogen to form plasmin, which dissolves fibrin in clots, thrombi, and emboli? + Sueptokinase * Streptococcus sanguis + Streptodomase + Prevotella intermedia + Hyaluronidase * Spirochetes. + Erythrogenic toxin + Actinomyces israeli + Streptolysin O + Porphyromonas gingivalis + Streptolysin S + Pyrogenic exotoxin A MICROBIOLOGY / PATHOLOGY Bact ‘Opsonization of microbial cells by complement is prevented by: MICROBIOLOGY / PATHOLOGY The synthesis of single-stranded RNA (ribonucleic acid) molecules based on a DNA sequence is called Bact + Peptidoglycans ‘+ Translation ++ The capsule ‘Transcription ‘Teichole acid ‘= Transduction * Adhesin Can 208-207 — ENA cas crores. 20 oan pecs = Transcription Transcription occurs in the cytoplasm of prokaryotes, while It occurs in the nucleus of ‘eukaryotes. The two-strands of the DNA must be unwound temporarily by DNA gyrase, allow- lng RNA polymerase to access the DNA for use as a template for RNA production. Three kinds ‘of RNA may be produced: (1) ribosomal RNA (rRNA), which combines with proteins to form ribosomes, (2) transfer RNA (IRINA), which transports amino acids to the ribosomes for assembly info proteins, and (3) messenger RNA (mRNA), witch dictates the sequence of ‘amino acid assembly. Important: Transcription is the synthesis of mRNA from DNA by DNA- dependent RNA polymerase, in other words, itis the synthesis of RNA from a DNA template. In bacteria itis mediated by an RNA polymerase, Translation is the process wherein information in the form of nitrogenous bases along an ‘mRNA is translated into the amino acid sequence of a protein ‘Transduction is the transfer of DNA via a phage particle. Does not require cell to cell con- tact. Reverse transcription is the formation of DNA from an RNA template, Retroviruses (2.9, HIV, tumor viruses), which are enveloped and contain a linear, single-stranded, positive- ‘sense RNA genome ulilized this process. They use their RNA genome as a template for an RNA-directed DNA polymerase. These viruses have a viron-associated reverse transcriptase, ‘which makes DNA copies from RNA. This DNA is then integrated into the host genome, "Note: This RNA-directed synthesis of DNA is the reversal of normal informational flow within the col. + Streptokinase (sometimes called fibrinolysin) Streptococcus pyogenes produces a wide array of virulence factors and a very large umber of diseases. Virulence factors of Group A streptococci include: (1) M protein, fibronectin-binding protein (Protein F) and lipoteichoic acid for adherence; (2) hyaluronic. ‘acid capsule as an immunological disguise and to inhibit phagocytosis; M-protein to Inhibit phagocytosis (3) invasins such as streptokinase, streptodomase (DNase B), hyaluronidase, and streptolysins; (4) exotoxins, such as erythrogenic toxin which causes the rash of scarlet fever and systemic toxic shock syndrome. Notes: 1, Streptodornase (DNase 8) depolymerizes DNA in exudates or necrotic tissue. 2, Hyaluronidase (spreading factor) degrades hyaluronic acid, which is the ground} substance of subcutaneous tissue. 3. Streptolysin O is a hemolysin that is inactivated by oxidation (itis oxygen-labile. I is also antigenic. ‘Streptolysin S is a hemolysin that Is not inactivated by oxygen (its oxygen-stabie) | Itis not antigenic. 5. Pyrogenic exotoxin A is a toxin similar to the staphylococcal toxic shock syndrome} toxin. 6. Exotoxin B is a protease that rapidly destroys tissue. 7. Erythrogenic toxin causes the rash of scarlet fever. +The capsule Phagocytes locate microorganisms through chemotaxis. They then adhere to the microbial cells, a process that is sometimes facilitated by opsonization, wherein the microbial cell is coated with plasma proteins. Pseudopods then encircle and engulf the microbe. The phagocytized microbe, enclosed in a vacuole called a phagosome, is usually killed by lysosomal enzymes and oxidizing agents Important: This coating of the organisms by molecules that speed up phagocytosis, is termed “opsonization”, and the Fc portion of antibody, and C3 are termed “opsonins”. ‘The capsule protects bacteria from phagocytosis in two diferent ways. Encapsulated bacteria are not readily phagocytosed because the simy capsules makes it dificult for phagocytes to hold firmly on the bacterial surface. Some of the microbial cell wall ‘components (complement receptors, for example) are masked by the capsule making it dificult or impossible for complement to bind. Encapsulated bacteria, therefore, cannot bind complement crectly. The plasma membrane of phagooytes possess receptors for €3b, C34, C3bi, or mannose. However, hese receptors of phagocytes cannot function ficiently In such encapsulated bacteria unless they are frst coated with specific ‘nticapsular antibodies and complement. All ther bacterial components do not mask the ‘complement receptors or opsonin receptors of the bacterial coll surface. + Prevotella intermedia + Spirochetes (specifically, intermediate-sized spirochetes) 'ANUG is an anaerobic infection of the gingival margins causing ulceration and, if left Untreated, destruction of the gingiva and underlying bone. The interproximal areas are affected first. Spirochetes invade the epithelium and connective tissue. Prevotella Intermedia, Fusobacterium species as well as Selenomonas species can also be seen in the necrotic zones superficial to the zone of spirochete infitration. Predominant subgingival bacteria associated with gingival health: + Streptococcus mitis and sanguis + Actinomyces viscosus and naeslundi + Rothia dentocariosus + Staphylococcus epidermidis + Small spirochetes. Note: IgG is the immunoglobulin that is found in the highest concentration in serum ‘samples from patients with periodontal disease. MICROBIOLOGY / PATHOLOGY MICROBIOLOGY / PATHOLOGY Bact All of the following are stages of phagocytosis except: During conjugation, what is transferred from the Hfr bacterium to the F~ bacterium? + Chemotaxis +The 80x factor (F factor) + Adherence + Portions ofthe Hfr chromosome + Transduction “The sex factor and portions of the Hfr chromasome + Poeudopodium formation + Nothing is transferred *Phagosome formation + Phago-lysosome formation ‘eo e2n8 2007 BEAL. OEE zorip 208-207 — BEAL DECKS MICROBIOLOGY / PATHOLOGY Bact All of the following statements concerning Streptococcus pyogenes are true except: MICROBIOLOGY / PATHOLOGY Which of the following bacteria are facultative anaerobic, gram-negative rods? Bact + Is also known as Group B beta-hemolytic Streptococcus and is not a frequent bacterial pathagen of humans + Escherichia + Mycobacterium + Itis a gram-positive coccus that occurs in pairs or chains, Its frequently part of the + Neisseria ‘endogenous microflora that colonizes the skin and oropharynx + Its the cause of several acute pyogenle (pus-forming) infections in man, such as scarlet fever, erysiplas, and sore throat (“strep throat’) + Bacteroides + Itelaborates several exotoxins (i¢., erythrogenic toxins and streptolysins S and 0) roi 020-207 —DaNTaL Deo += Transduction Phagocytosis is mediated by macrophages and polymorphonuclear leukocytes. It involves the ingestion and digestion of the following: microorganisms, insoluble particles, ‘damaged or dead host cells, cell debris and activated clotting factors. There are several stages of phagocytosis: 2. Adherence -> this works reasonably well for whole bacteria or viruses, but less 80 for proteins or encapsulated bacteria. 3. Pseudopodium formation -» this is protrusion of membranes to flow around the ‘prey’. 4. Phagosome formation -> fusion ofthe pseudopodium with a membrane enclosing the “prey’ leads tothe formation of a structure termed a phagosome. ‘5. Phago-lysosome formation -> the phagosome moves deeper into the col, and fuses ‘with a lysosome, forming @ phagolysosome. These contain hydrogen peroxide, active oxygen species (ree radicals), peroxidase, lysozyme, and hydroytc enzymes. This leads to the digestion of the phagolysasomal contents, after which they are eliminat- ed by excoytosis Phagocytes: + Fixed > do not circulate, fixed macrophages and cells of the reticuloendothelial sys- tom. + Free > circulate in the bloodstream, include the leukocytes and the free macrophages. + Escherichia ‘Many important pathogens, including Enterobacteriaceae (Escherichia, Salmonella, Shigella, Kiebsiola, Yersinia, Enterobacter), Virionaceae (Vibrio), Pasteurellas (Hemophitus, Gartinerell, and Pasteurella), belong to the group of facultative anaerobic, gram-negative rods. Note: These bacteria are highly invasive and have the ability to readily become resistant to antibiotics Important: Escherichia coll (E.col) in not usually considered a pathogen. However, it can be responsible for unary tract infection, dlarrhea, and very serious foodborne diseases. Groups of bacteria: + Spirochetes -> Treponema, Borrelia + Helical / vibroid gram-negative bacteria -> Spirilum, Campylobacter, Hellocobacter sgative aerobic rods and cocci -> Pseudomonas, Bordetella, Neisseria, Legionella + Gram-negative anaerobic rods > Bacteroides, Fusobacterium, Prevotella, + Rickettsiae and Chlamydiae -> Rickettsia, Coxiella, Chlamydia + Mycoplasmas -» Mycoplasma, Ureaplasma + Gram-positive cocci -» Stephylacoceus, Streptococcus, Enterococcus, Peptostreptocaccus + Endospore forming rods and coce! -> Bacillus, Clostridium + Regular non-sporo forming gram-positive rods —> Lactobacillus + Irregular non-spore forming gram-positive rods —> Corynebacterium, Actinomyces + Mycobacteria -> Mycobacterium + Actinomycetes -> Streptomyces, Nocardia, Rhodococcus, + Itis also known as Group B beta-hemolytic Streptococcus and is not a frequent bacterial pathogen of humans “--This is false; its also known as Group A beta-hemolytic Streptococcus and is one of tho ‘most frequent bacterial pathogens of humans. ‘The diseases caused by streptococcus pyogenes may be considered in two categories: 41. Suppurative + "Strep throat” -> commonly affects children and adolescents + Scarlet fever -> caused by erythrogenic toxin + Erysipelas — acute contagious disease marked by a circumscribed red eruption on the skin, chills, and fever + Impetigo “> a localized, intragpidermal infection of the skin seen in preschool-aged children + Cellulitis > results from traumatic inoculation 2. Nonsuppurative + Rhoumatic fever -> usually begins witn a sore throat and after several days thore is a ‘rapid rise in temperature, prostration, and inflammation of the joints; the heart is often affected, + Acute poststreptococcal glomerulonephritis -> occurs primarily in children; symp- toms include increased fluid retention, dark tos-colored urine, and elovation ofthe BP. Note: Group 8 Streptococcus (S. agalactiae) is the leading cause of neonatal pneumoni ‘meningitis, and sepsis. + Portions of the Hfr chromosome Conjugation is a form of sexual reproduction in which DNA is transferred from one live bacterium to another through direct contact. This physical contact is established through the presence of pil. Note: In this process, the greatest amount of genetic infor- mation is transferred from one cell to another (compared to transduction and transfor- mation). F factors are plasmids transferred from a donor cell (an F*cell) to a recipient cell (an F-cel) during conjugation. ‘An Hie (high frequency of recombination) isa cell with an F plasmic ‘chromosome. \corporated into the: MICROBIOLOGY / PATHOLOGY Vac bacterial toxin that has been weakened unti itis no longer toxic but is strong enough to induce the formation of antibodies and immunity to the specific disease caused by the toxin is called a (an): + Antitoxin + Toxoid MICROBIOLOGY / PATHOLOGY Vac Which disease is provonted with a vaccine that contains attenuated live viruses? + Hepatitis A + Varicella (Chickenpox) + Influenza + Rabies caprin ca 07 — pena peo MICROBIOLOGY / PATHOLOGY Bact The symptoms of sepsis include all ofthe following except: + Fever + Weakness + Painful urination “Nausea + Vomiting + Diarrhea = Chills eoriatezes- 2007 DOA. DECKS MICROBIOLOGY / PATHOLOGY Vac Freund’s adjuvant is a mixture composed of all of the following except: * Mineral oll * Lanolin + Formalin + Inactivated and dried mycobacteria rm oz 007 — NAL OE + Varicella (Chickenpox) Attenuated live virus Attenuated live virus Purified HBsAg: Recombinant HBsAg. Attenuated live virus Attenuated live virus Attenuated live virus Attenuated live virus Sabin)-oral Inactivated virus Salh)-injection Inactivated virus Inactivated virus Attenuated live virus Inactivated virus Influenza Rabies Varicella (chickenpox) Hepatitis A + Toxoid (also called immune globulins) Toxoids are inactivated protein exotoxins that induce the formation of specific antitoxin antibodies that serve as the basis for the specific protection from the toxin. ‘These vaccines are useful against diphtheria, tetanus, and other diseases. These toxoids are prepared by treating toxins with formaldehyde. Note: Not all toxins can be converted to toxoids by treatment with formaldehyde, Antitoxin is an antibody formed in response to a specific toxin. A serum containing antitoxins can be used for either the treatment or prevention of certain bacterial diseases. The antitoxin can neutralize unbound toxin to prevent the disease from progressing. Tetanus antitoxin is used both in the treatment of tetanus and in its prevention (prophylaxis). Botulinum antitoxin is used in the treatment of botulism. Diphtheria antitoxin is used in the treatment of diphtheria, Note: As with other inactivated vaccines, there are disadvantages with toxoid vaccines. Even with an adjuvant added, these vaccines do not produce a full immune response. Booster shots are needed to maintain the immunity + Formalin Fround's adjuvant is an antigen solution emulsified in mineral oll and lanolin, used as ‘an immunopotentiator (booster of the immune system). The so-called complete form (FCA) is composed of inactivated and dried mycobacteria, usually Mycobacterium tuberculosis. The so-called incomplete form (FIA) is the same adjuvant, but without the mycobacterial components. This adjuvant is used to elicit stronger T- and B-cell mediated responses when antigens alone do not evoke sufficient immunogenic responses. The role of adjuvants: + Used to enhance antibody response + To enhance the uptake of the antigen by antigen-presenting cells (0.0., macrophages) + They are added to vaccines to slow down the absorption and increase the effectiveness of the vaccine Example: When protein antigens are mixed with aluminum compounds, a precipitate is formed that is more useful for establishing immunity than are the proteins alone. ‘Alum-precipitated antigens are released more slowly in the human body, enhancing the stimulation of the immune response. The use of adjuvants eliminates the need for repeated booster doses of the antigen and permits the use of smaller doses of the antigen in the vaccine. + Painful urination ‘Sepsis is @ severe illness caused by overwhelming nection af the bloodstream by toxn-producing bacteria, Sepsis is caused by bacterial infection that can orginate anywhere inthe body. Common siles include the folowing: +The kidneys (upper urinary tract infection) me liver or the gall bladder +The bowel (usually seen with peritonis) +The skin (colts) ‘The lungs (bactarial proumonia) ‘Meningie may also be accompanied by sepsis. In children, sepsis may accompany infection af the bone (osteomyelitis). In hospitalized patients, common sites of infection inchide intravenous lines, surgical wounds, surcical drains, and sites of skin breakdown known as decubitus ulcers or bedsores, In some cases, sepsis leads to a life-threatening condition called septic shock. The ‘main culprits of sepsis seem to be Staphylococcus aureus, Escherichia coli, and Klebsiolla. In ‘dalton, the LPS (endotoxin) released from the walls of dead gram-negative bacteria when they ‘are lysed can cause septic shock as well as complement-actvaied anaphylactic shock, Bacteremia refers to the presence of viable bacteria in the circulating blood. This can occur even Inhealthy individuals. For example, folowing some dental procedures, such as an oral prophylaxis ‘and extractions, bleeding of the gums results ina transient systemic bacteremia. Clinical signe and symptoms aro usually not prosont. ‘Viremia is defined as a viral infection of the bloodstream. Its @ major feature of disseminated Infections. The infecting vius is very susceptible to circulating antivodies, MICROBIOLOGY / PATHOLOGY MICROBIOLOGY / PATHOLOGY Bact Vac Human HBIG (human serum containing a high titer of antibodies against HBV) to prevent hepatitis B in those not actively immunized with the HepB vaccine is an example of The erythrogenic exotoxin produced by Streptococcus pyogenes causes: * Gas gangrene + Bacterial dysentery + Naturally acquired passive immunity + Naturally acquired active immunity + Scarlet fever Tetanus + Artificially acquired active immunity + Artificially acquired passive immunity MICROBIOLOGY / PATHOLOGY Vac ‘The streptococcus pneumonia vaccine is an example of a MICROBIOLOGY / PATHOLOGY “Microorganisms which grow at an optimum pH well above neutrality (7.0) are called: Bact + Acidophiles * Capsular polysaccharide vaccine + Inactivated protein exotoxin (toxoid) vaccine + Alkaliphiles + Killed bacterial vaccine + Neutrophiles + Live attenuated bacterial vaccine ror o08- 2097 — gna OOK cam e286 207 — era es + Alkaliphiles “The range of pH over which an organism grows is defined by three cardinal points: the min- imum pH, below which the organism cannot grow, the maximum pH, above which the ‘organism cannot grow, and the optimum pH, at which the organism grows best. ‘Those which grow best at neutral pH (7.0) are called neutrophiles. Most bacteria are con- sidered to be neutrophiles. Examples include: Pseudomonas aeruginosa, Clostridium ‘sporogenes, and Proteus species. ‘Those which grow at an optimal pH well below neutrality (7.0) are called acidophiles. Examples include: Thiobacilus thiooxidans, Sulfolobus acidocaldarius, and Bacillus aci- docaldarius. Note: Obligate acidophiles, such as some Thiobacillus species, actually require @ low pH for growth since their membranes dissolve and the cells lyse at neutrality. ‘Those that grow best under alkaline conditions are called alkaliphiles. Examples include: Nitrobacter species and Streptococcus pneumoniae. Remember: Aciduric means that bacteria is capable of or can tolerate ving under acid conditions, Acidogenie moans that bacteria can produce relatively high concentrations of ‘acid (Streptococcus and Lactobacillus produce lactic acid as the primary fermentation prod- uct). Streptococe are the primary acidogenic microorganisms in the oral cavity. Lactic acid is the main cause of enamel decalcification. + Scarlet fever ‘Note: Enterotoxins are toxins that are specific for the cells of the mucous membrane of the Intestines. An erythrogeniec exotoxin le a toxin that produces a rash or reddening ofthe skin. eee ener Perera Secs __ | pisens Cloatidium [Several nevrotonine [Botulism Paralysis; Blocks neural ansmiseion osama peingens | toxin fa collagenase) breaks down fibrous uasve ‘cioaiiue |Neurotoxin FFeanwe Spas paralysi: interferes with eta cts) tor neon [Connebaciriom [Diphtheria toda [Diphtheria ioerine Strepioaoceus Various dresser pyopenee “iekoumatie fever |Caumor symptoms of heumatie fever romp \Seremis = Searettever [Causes rash of sare fever eae Bacerat dysentery [Homrhage, paralysis Staphylococcus Food polaoning [ison iflamamaton + Capsular polysaccharide vaccine ‘Capeular polysaccharide vaccines (these vaccines contain polysaccharide capsular material from the bacteria, usually canjugeted to protein for greater immunogenic): + Sireptococcus pnaumonia vaccine (for pneumonia) + Neisseria meningides vaccine (for meningitis) + Haemophilus influenzae vaccine (for meningitis) ‘Toxold vaccines: + Corynebacterium diphtheriae vaccine (for diphtheria) + Glostiaium tetanl vaccine (for tetany) Killed bacterial vaccines: + Bordetella pertussis vaccine (for whopping cough) + Salmonelia typhi vaccine (for typhoid fever) + Vibro cholerae vaccine (for cholera) Live attenuated bacterial vaccines: + Mycobacterium bovis vaccine (for TB) + Francisella tularensis vaccine (fr tularemia) + Coxiella burt vaccine (for O fever) ‘Active immunity is induced by vaccines propared from bacteria or their products. Passive Immunity Is provided by the administration of preformed antibody in preparations called immune ‘lobuling (oxo). Passive-active Immunity involves giving both immune globulins (toxins) to Provide immediate protection and a vaccine to provide long term protection. + Artificially acquired passive immunity Remember: Acquired immunity occurs naturally and artificially. It can be active or passive. 41. Naturally: + Active -> person Is exposed to an antigen and the body produces antibodies. For ‘example, recovery from an infection with mumps virus confers lifelong immunity. + Passive -» antibodies (IgG) passed from mother to fetus during pregnancy and IgA passed from mother to newborn during breast-feeding. ically: + Active -> vaccination with kiled, inactivated, or attenuated bacteria or toxoid. + Passive — injection of immune serum or gamma-globulin. Important: The purpose of the rabies vaccine and tetanus vaccine (examples of Artificially acquired active immunity) is to elicit an immune response before the onset Of disoase symptomatology. The effectiveness of this type of vaccine depends on the relatively slow development of the infecting pathogen prior to the onset of disease ‘symptoms and the ability of the vaccine to initiate antibody production before the active toxins are produced and released to the site where they can cause serious disease symptoms. MICROBIOLOGY / PATHOLOGY Lactobacillus is a part of the group of: + Irregular, nonsporing, gram-negative rods ‘+ Regular, nonsporing, gram-positive bacteria = Anaerobic, gram-negative cocel + Endospore-forming, gram-positive cocci MICROBIOLOGY / PATHOLOGY Bact A gelatinous coat that is often used as an indicator of virulence is called the: + Call wall + Capsule + Pellicle + Plasma membrane MICROBIOLOGY / PATHOLOGY Bact {All of the following bacteria have been found to be the principal bacteria associat- ed with aggressive periodontitis except: + Actinobacillus actinomycetemcomitans (AA) + Capnocytophaga ochraceus + Wolinlla recta + Provotela intermedius + Eikenella corrodens MICROBIOLOGY / PATHOLOGY Bact A thin murein layer is characteristic of: + Gram-positive bacterial call walls + Gram-negative bacterial call walls * Vial cell walls + Fungal cell walls + Capsule ‘The capsule surrounds the cell wall of certain bacteria and is especially important in protecting these cells against phagocytosis by eukaryotic cells. The presence of a Capsule can be a major factor in determining the pathogenicity of a bacterium, that i, the blity of a bacterium to cause disease in the organism that it infects, ‘These antiphagocytic polysaccharide capsules surrounding the cells of strains of streptococcus pneumoniae, for example, permit these bacteria to invade the normal {defense mechanisms of the host, allowing them to reproduce and cause the symptoms ‘of pneumonia. The virulence of other bacteria, including Haemophilus influenzae, Klebsiella pneumoniae, and Cryptococcus neoformans is also enhanced by capsule production. ‘The cell membrane (cytoplasmic membrane) is a selectively permeable membrane that is involved in energy transformations (ie., oxidative phosphorylation). It is bordered ‘externally by the cell wall in most bacteria. The cell wall surrounds the plasma mem- ‘brane and serves to protect the call from changes in osmotic pressure, anchor flagella, ‘maintain cell shape, and control the transport of molecules into and out of the cell. The plasma membrane is a dynamic, selectively permeable membrane enclosing the eyto- plasm. Its located between the call wall and the cytoplasm, and it regulates movement Of substances, including water, into and out ofthe cell. + Regular, nonsporing, gram-positive bacteria Most species of this non-sporeforming bacterium ferment glucose info lactose, hence the name LLactobacilus. The most common applicatlon of Laciobacalus is industrial, spectically for dry production. This gonus also contains several bacteria thet make up part of the natural Nora of the human vagina. Because of their ably to derive lace: acid ftom glucose, these bactora create an acidic environment ‘hich inhibits growth of many bacteral species which can laa! o urogenital nfectons, though Lccbacts paces ere nama preset in io numbersn ha ral avy hey ar Roe Tend association wih dara cares peciay Lacoecus css, mos oban ae sana Lactobacius acidophilus is added to commercial mik products 1 exsist lactose intolerant individuals ‘in aigesting lactose sugars. The enzymes produced by these bacteria convert mik sugars to products, that do not cause Gi problems, Lactic acid bacteria incude Lactabecius and Streptococcus. These bacera use the lactic acts fermen- ‘ation pathway in which pyrivaie Is reduced to lactic acid. These two acta ate lso referred (0 a9 ‘ciduri, meaning that they ean tolerate an acid environment, and acidogentc, meaning aid forming, Rote: }.Lectobaci generally harmles churns, rae ining hamid iectons or dseases Tost ‘ment of this vancomyclnresistant microbe usualy consis of high doses of pericin in combat mete ity igh penicl binato 2. Streptococcus mutans isthe main culprit in dental cries (especialy smooth surface cares) ‘Actinomyces has been found fo be a causative agent of root surface cari + Gram-negative bacterial cell walls. Murein or peptidoglycan is the backbone biochemical of the bacterial coll wall it is ‘composed of repeating alternating units of N-acetylglucosamine and N-acetyimuramic ‘acid and has cross-inked, short peptide chains, some of which have unusual amino acids. Gram-positive cell wal + Thick murein layer that makes up approximately 90% of the cell wall + Teichoic acids, polysaccharides that serve as attachment sites for bacteriophages Gram-nogative coll wall: + More complex (muttilayered) than gram-postve coll wall + Thin murein layer that accounts for only about 10% ofthe coll wall, + Lipoproteins are an integral part of the cell wall + Lipopolysacchaide layers (endotoxin is equated with this component) + Phospholipids + Proteins The proteins, lipopolysaccharide layers, and phospholipids make up the cell envelope of the gram-negative bacterial cell. This outer membrane protects the cell from antibi- tics (0.9., penicilin) and enzymes (0.2, lysozyme). + Wolinella recta Aggressive periodontitis can be further classified on the basis of extent and severity: 1. Generalized — Prevotella intermedius and Eikenella corradens predominate. It ‘occurs between the ages of 12-25 and is characterized by rapid, severe periodontal destruction around most teeth. It appears to be associated with systemic diseases such as diabetes mellitus Type 1, Down syndrome, neutrop- conias, Papillon-Lefevre syndrome, and different types of leukemias. 2. Localized -> gram-negative anaerobes Actinobacillus Actinomycetemcomitans (AA) and Capnocytophaga ochraceus predominate. Prevotella intermedius and Eikenella corrodens may also be present to a lesser extent. Localized aggressive periodontitis occurs in an otherwise healthy adolescent (12-19). It is character- zed by severe periodontal destruction around either the maxillary / ‘mandibular first molars or the maxillary / mandibular anterior teeth. The one outstanding negative feature Is the relative absence of local factors (pleque) to explain the severe periodontal destruction which is present. Possible etiologic factors include a genetic predisposition or a dysfunction of neutrophils (a chemo- tactic defect). Note: AA and Capnocytophaga ochracous are also associated with periodontitis in juvenile diabetes. MICROBIOLOGY / PATHOLOGY Bact The acid-fast stain may be performed on any clinical specimen but is most ‘commonly used in examining sputum for which bacteria? MICROBIOLOGY / PATHOLOGY Bact Which of the following bacteria form unusual acids, called mycolic acids, that are associated with their cells walls? + Escherichia coli * Streptococe! * Streptococcus pneumoniae + Lactobaeili * Clostridium botulinum * Mycobacteria * Mycobacterium tuberculosis + Staphylococci rp e206 2091 NTA DECKS Cenripteaee 207 Bawa. DECKS MICROBIOLOGY / PATHOLOGY Which bacteria is a gram-positive coccus that grows in grape-ike, kregular clusters in culture? MICROBIOLOGY / PATHOLOGY Bact Hyaluronidase is an enzyme produced by bacteria that digests connective tissue, allow- ing them to spread through tissues more readily. Species from which bacteria below do not produce this enzyme? + Streptococcus * Streptococcus + Neisseria + Clostri m + Salmonella * Staphylococcus + Staphylococcus + Neisseria + Staphylococcus ‘Staphy/ococcal infectons are suppurative infections usually caused by S. aureus. Examples of such Infections include abscesses, endocarditis, impetigo, osteomyelitis, pnaumonia, and septicemia, S. ‘aureus may cause acute-onsat food poisoning (due fo enterotoxlns, which ar toxins specifi or cis ‘ofthe intestine). Protsin Aisa component of th cell wal of Staphyiococcus that may be responsible for its virulence, ‘Staphylokinase (produced by S. aurous), Streptokinase (produced by hemolytic strepiococa), and Urokinase are enzymes thal cleave plasminogen, producing plasmin, which causes the iquefscton of fiir. They ae used inthe renova of blood cots. Romomber:Stroptococe! are aerobic to facultatvely anaerobic gram-positive coca that grow in pals or chains in cute. They are the most numerous group of microorganisms tht occur inthe oral cavity, \wiere they can grow and cause dental earles (rainy S. mutans). Othor moro serous ifocions causod by Streptococcus inéude pneumonia (S. pneumoniae), rheumatic fever (S. pyogenes), and heart valve infections (S. vindens) “The genus Neisseria contain two important human pathogens, N. gonorhoeae and N. meninghiss. N. ‘gonorthoage causes gonerthea, and N. meningitd isthe cause of meningococcal meningitis. N. gon- ‘orrhoeae infections have a high prevalence and low mertalty, whereas N, maningitisinfectons have 2 low prevalence and high mortality Salmonelia is a gram-negative facukatve ro shaped bacterium in the same proteobactoral family as Escherichia col, the farily Enterobactoiacoae. tivialy known as "enterle” bacteria, In humans, ‘Saimonola aro the cause of two disoasee called salmonellosis: entre fever (typhoid), resulting from bacterial invasion of te bloodstream, and acute gastroenteritis, resulting ftom a foodborne infection J intoxeation. + Mycobacterium tuberculosis ‘The acid-fast smear plays an important role in the early diagnosis of mycobacterial Infections. Microscopy isthe oldest, easiest, most rapid, and inexpensive procedure that can 'be performed in the laboratory to detect the presence of acid-fast bacil, ‘The acid-fast staining is ono of the methods used to diagnose active tuberculosis. It is @ ‘method of staining used in bacteriology in which a smear on a slide Is flooded with carbol- {fuchsin stain, decolorized with acid alcohol, and counterstained with methylene blue. Acid- {ast organisms resist decolorization and appear red against a blue background when viewed Lunder a microscope. This property of being acid-fast is atributable to the presence of lipids ‘and waxes (mycolic acids) in the cell wall of certain bacteria, ‘The classic skin test (PPD skin test) Is another method of testing for tuberculosis. It may indicate an infection, but not whether the infection is active. A purified protein derivative (PPD) extract from mycobacterium tuberculosis is Injected subcutaneously, and the area ‘hear the injaction is observed for evidence of a delayed hypersensitivity reaction. A pos- ve test indicates a hypersensitivity to tuberculoproteins. Important: Mycobacterium tuberculosis is an acid-fast, obligate aerobic, niacin-producing bacterium, which grows slowly (it has @ generation time of 1 to 6 hours) on the Lowenstein- Jensen or other atifcial culture media. It produces neither exotoxins or endotoxins. This bactetia is also capable of withstanding severe desiccation and stil remains infectious. + Nolsseria Hyaluronidese [Breaks down hyaluronic acid_| Various species of Stepiococeus, spreading) Staphylococcus, and Clostridium [Conguiase [Convers fibrinogen to fibrin | Staphylococcus aureus red teed ea PPhoephotipase [Ly Staphilococeusaureue esihinase | Desuoys red blood cells and | Clostridium perfingens other ate celle [Cottgenase [Breaks down collagen Clostiaium sp, Bacteroides sp, Actinobe- (comective disse fiber) illus actinomyeetemeomitans, and Bacil- us sp [Fibrinoysins: [Dissolve blood clos Staphylokinase Staphylococcus Strepiokinase Streptococcus \Note: Hyaluronic acid is an extremely hydrophilic glycosaminoglycan, and is found in nearly ‘every cell in the body. The molecule is made up of a repeating sequence of two modified ‘simple sugars, one called glucaronic acid and the other N acetyl glucosamine, Itis the major ‘component of hyaline cartilage “+ Mycobacterla ‘These mycolic acids are localized in the inner leaflet of the mycobacterial cell wall either ‘covalently bound or loosely associated with arabino-galactan polymers. The mycolic, acids impede the entry of chemicals causing the organisms to grow slowly and be more resistant to chemical agents and lysosomal components of phagooytes than most bac- teria. Remember: Isoniazid inhibits mycolic acid biosynthesis and is an efficient antimy- cobacterial agent. ‘Mycobacterium are gram-positive, nonmotile, rod-shaped bacteria Examples include: + M. tuberculosis -> the causative agent of tuberculosis. *M. leprae — the causative agent of leprosy (also called Hansen's bacilus). ‘Notes: 1. A tubercle or Ghon focus is a smal, rounded nodule produced by infection with M. tuberculosis. It is the primary lung lesion of pulmonary TB. [2. Mycolic acids are also present in the cell walls of actinomycetes. MICROBIOLOGY / PATHOLOGY MICROBIOLOGY / PATHOLOGY Endotoxins are the lipopolysaccharide component of the call wall of: Bact Endotoxins are: + Gram-positive bacteria ‘+ Produced by gram-positive bacteria and are located in the inner membrane of the bacteria + Gram-negative bacteria + Produced by gram-negative bacteria and are located in the inner membrane of the + Both gram-positive and gram-negative bacteria bacteria * Produced by gram-negative bacteria and are located in the outer membrane of the bacteria * Produced by gram-positive bacteria and are located in the outer membrane of the bacteria ‘ey e208 — TAL OFCKS ri 20-2007 — BETA DEORE MICROBIOLOGY / PATHOLOGY Gamma-hemolytic streptococci can be differentiated from alpha and beta streptococci on blood agar plates by observing: MICROBIOLOGY / PATHOLOGY Bact Identity the following phases labeled A —D below of a standard bacterial growth curve. Bact + Aclear zone of hemolysis about the colony with no intact corpuscles + Small colonies surrounded by a greenish discoloration, + No hemolysis + Small colonies surrounded by a zone of partial hemolysis and an outer clear zone Center 2007 eras BEEK aig 020-2007 — BOA. DEKE + Gram-negative bacteria Because the lipopolysaccharide (LPS) is part of the bacterial cell structure, it is called endotoxin. Although all gram-negative bacteria have LPS in their cell walls, LPS is not toxic unless it is released from the cell. The death of these bacterial cells causes their walls to disintegrate, releasing the LPS toxin. These endotoxins exert their pathologic effects by activating the complement cascade. [Gram-positive and gram-negative [Gram-negative [Composition Proteins Tipopalysaccharides [Released by organisms | Yes No. Toxoids (accines) ves No. Degree Very potent Less potent Specificity for target ces [High Low + No hemolysis, Some streptococcal species produce toxins, called hemolysins, that cause lysis of ‘erythrocytes. Whan these bacteria are grown on blood agar plates, zones of clearing may be ‘seen around the bacterial colonies. Alpha-hemolytic streptococe! produce a zone of incomplete hemolysis and green ‘discoloration adjacent to the colony. Beta-hemolytic streptecocel produce a clear zone of hemolysis around the colony. Gamma-Streptococel produce no hemolysis. Note: Oral streptococci ae usually alphachemolytic. Beta-nemolytic forms are classified into groups (A, B, ©, etc.) according to the carbohydrate {ound inthe cell wal ‘Group A strains are pathoganic for humans. They are further subdivided by Arabic numerals Into specific antigenic types based on the cell wall M protein. This M protein seems to be ‘closely associated with the virulence of the bacteria. The prototype is 8. pyogenes (causes ‘theumatic fever, scarlet fever, and sore throat) Toxins produced by Group A beta-hemolytic streptococe! include erythrogenic toxin, deoxyribonuciease, hemolysins (streptolysins © and S), hyaluronidase, streptokinase, and strepiodomase. Important: Streptococcal pyrogenic exotoxin A (SpeA) is a superantigen produced by ‘Streptococcus pyogenes and Is associated with severe infections characterized by rash, hypotension, multiorgan failure and a high mortality rate. A= Lag phase B = Log (logarithmic or exponential) phase C= Stationary phase D = Decline (death) phase ‘When bacteria are placed in fresh, nutrient-rich me phases of population growth: ‘Al Lag phase -+ cells are metabolically active but are not dividing. This is a period when the cells are resynthesizing enzymes, coenzymes, etc., necessary for growth and division, 8) Log phase (exponential growth phase) -» bacteria are growing and dividing at an ‘exponential, or logarithmic, rate. This is the period of fastest growth: the generation time Is maximal and constant. Note: Most cidal antibiotics work best during this phase. This is the best phase to stain bacteria culture if you want to study them, | Stationary phase —> at this point, the medium is becoming depleted in some nutri. entsand toxic quantities of waste materials may be accumulating. The number of new cells produced is offset by the number of cells that are dying; thus, the total num- ber of viable cells remains constant. D] Death phase -> conditions are becoming less and less conducive to cell growth. Cells are dying more rapidly than new ones are being formed, resulting in a logarth- mic decrease in the number of cells. am, they exhibit four characteristic + Produced by gram-negative bactoria and are located in the outer membrane ofthe bacteria “The lipid component ofthe tipopolysaccharides (LPS) i the endotoxin. Endotoxns are not secreted by ‘bacteria calls. The bacterial ell mist die and the outer membrane be broken down for the endotoxin 10 'be released into the bloodstream. The hosts responses to endotoxins include chil, fever, weakness, gon- eralized aches, and, in severe cases, shock and death Endotoxins are highly potent lipopolysaccharides released trom the col walls of gram-nogatve bacte- Fa, Mince amounts nthe oral cosa cause inflammation and resorption of adjacent bone. Endotoxin has 2 chemotactic effect on neutrophilic granulocytes and induces phagocytoss by tese cels. {ound in diseased periodontal Ussue is subgingival plaque. CCollagenasa isthe protease which degrades colagan, one ofthe body's primary connective issues In pation with periodontal diseaso, the collagen which forms the structural basis of tho periodontu is broken down by colagenase. This protease has been demonstrate tobe e pat of the component system Inthe folowing bacteria: Porphyromonas species, Clostrchum species, Bacteroides species| and AA ‘Although most bacterial exotoxns are proteinacoous in nature, endotoxin is a lpopctysaccharid} ‘complex composed of ipl A (portion most responsible for tox activity) core polysaccharide, and 0" antigenic side chain, Endotoxin can activate the complement systom via the altematve pathway. C3 can be activated by endotoxin inthe absence of preceding activation of C1,4, and 2. As a result, the various com ploment components (C3, 5-9) are consumed and then ther actvty dlsappaars is reduced fom MICROBIOLOGY / PATHOLOGY Bact MICROBIOLOGY / PATHOLOGY All of the following statements concerning Staphylococcus aurous are true except: Members of which species below are predominant in saliva? + Actinomyces + Itis a gram-negative coccus that typically grows in a linear fashion and is a very uncommon bacterial pathogen + Veillonelta “Its the most common cause of suppurative infections involving the skin, joints, and bones and is the leading cause of infective endocarditis. + Streptococcus + Staphylococcus + Ibis coagulase positive whereas other Staphylococci are coagulase negative ‘It possesses a surface protein (protein A), that binds the Fe receptor of 196, thereby blocking complement activation by the classical pathway MICROBIOLOGY / PATHOLOGY MICROBIOLOGY / PATHOLOGY Disord Ie How long does it take to kill bacterial spores when a dental instrument is placed in a 2% solution of glutaraldehyde? Nourofibromatosis is a (an): + Sexlinked dominant disorder + 10 minutes + Autosomal recessive disorder 1h : '* Sex-linked recessive disorder + 10 nous + Autosomal dominant disorder + 26 nous ‘coy 2007 BETA. BEERS crore nee 200 — TAL BOS +10 Hours 2% glutaraldehyde is an alkalizing agent highly lethal to essentially all microorganisms If sufficient contact time (10 hours) is provided and there is absence of extraneous ‘organic material. Note: Alcohols, chlorhexidine, and quatemary ammonium compounds are disinfectants. ‘Advantages of glutaraldehydes: * Most potent category of chemical germi * Capable of kiling spores (after 10 hours) + EPA registered as an immersion steritant * Can be used on heat sensitive materials Disadvantages of glutaraldehydes: * Long period required for sterilization + Allergenic. ‘Not an environmental disinfectant + Extremely toxic to tissues Note: In hospitals, glutaraliehydes are used to sterilize respiratory therapy equipment. + Itis a gram-negative coccus that typically grows ina linear fashion and is a very ‘uneommen bacterial pathogen ‘This is false; it's a gram-positive coccus that typically grows in grape-ike clusters and is ‘one of the most common bacterial pathogens. Staphylococcus aureus cannot invade through intact skin or mucous membranes, and infection Usually begins with traumatic Inoculation of the organism. Once inside the body, it secretes a number of enzymes and toxins thet harm most tissues. Note: The coll wal ofS. aureus contains ribtol phosphate teichoic acd 'S. aureus infection usually produces suppuration and abscess formation. It most commonly ‘causes skin infections. Some examples include bolls, carbuncles, and slyes; scalded skin ‘syndrome and toxic shock syndrome; osteomyelitis; infecions of burns or surgical woun respiratory tract infections; septicemia; bacterial endocarditis; and staphylococcal food poisoning. Coagulase-negative staphylococe! (ieee virulent than S. aureus): + S. epidermis -» the most frequent cause of infections associated with medical devices. + 8. saprophyticus -> the frequent culpt of acute urinary ract infections in young women. Romember: Stophylococei ere facultative anaerobes that grow by aerobio respiration or by Termentation that yields principally lactic acid. The bacteria are catalase-positive and ‘oxidase-negative. S. aureus can grow al a temperature range of 15 to 45 degrees and at NaCl ‘concentrations as high es 15 percent. Nearly all strains of 8, aureus produce the enzyme coagulase. + Autosomal dominant disorder ‘Neurofibromatosis (sometimes known as Elephant Man's disease) is an autosomal dominant disorder that affects the bone, the nervous system, soft tissue, and the skin. it causes the {ormation of multiple, pedunculated, soft tumors (neurofibromas), and café-au-lait spots. ‘Two major subtypes exist: Neurofibromatosis Type 1 (NF-1), von Recklinghausen rneuroflbromatosis), which is the most common subtype and Is referred to 9s peripheral NF, ‘and Neurofibromatosis Type 2 (NF-2), which is roferred to as central NF. INF-1 is long lasting (chronic). it mostly affects nerves of the outer parts of the body (peripheral nervous system). Symptoms begin at birth or early in life. People with NF-1 Usually develop: ‘ Multipte birthmarks -» six or more light brown-colored birthmarks (café-au-ait spots) may be located anywhere on the body including the mouth and tongue. + Multiple neurofibromas -» a person with NF-1 may have any number of neurofibromas = from none to hundreds. The tumors are usually small, painless and slow growing. + Liseh nodules in the eyes -> small brown tumors (Lisch nodules) often appear on tho Colored part of the eye (ins) in peopie with NF-1 NF-2 (also known as bilateral acoustic neurofbromatosis), Is less common and mostly affects the central nervous system, causing tumors of the brain and spinal cord. Hearing loss that begins in the teens or early 20s is offen the first symptom. People with NF-2 Usually develop: + Aucitory nerve tumors + Other complications -» affected people may also have ringing in the ear(s), headaches, facial pain / numbness and trouble balancing, + Streptococcus Members of the “viridans” group of Streptococci (saiivarius, mitis, and sanguis) are predominant in saliva, Gram-positive filaments such as Actinomyces and gram- negative organisms such as Veillonella are also present, Oral cavity (saliva, tongue, plague) Streptococcus, veillonella, bacteriodes,fusobacterium, ‘eptostreptococcus, and actinomyces Lactobacillus, streptococcus, clostridium, veilonella, bacteroides, fusobacterium, escherichia, proteus, Klebsiella, and enterobacter Streptococeus, staphylococcus, moraxella, neisseia, hae- mopbilis, bacteroides, and fusobacterium | Gastrintestinal tract Upper respiratory tract (nasal cavity and nasopharynx) Lower respiratory wat, None Upper urinary wet hone tiineys and bladder) Genitourinary tact Steprococeus, lactbacilis, bacteroide, and lostrdium rethra and vaginal racts) MICROBIOLOGY / PATHOLOGY Ic MICROBIOLOGY / PATHOLOGY Ic Which of the following is a powerful oxidizing agent that inactivates bacteria and most viruses by oxidizing free sulfhydryl groups? Quaternary ammonium compounds, which are widely used for skin antisepsis, are classified as: + Alcohol = Nonionic detergents * Chlorine * Anionic detergents + Formaldehyde * Cationic detergents, * Phono! rp ez- 2001 — BBA. BECKS eo e208 - 2007 ITAL DEORE MICROBIOLOGY / PATHOLOGY MICROBIOLOGY / PATHOLOGY Ie The proper time and temperature for autoclaving is: + 950°F (177°C) for 1 hour ‘The term “antiseptic” BEST relates to a (an): + Autoclave + 250°F (121°C) for 15-20 minutes Dey tent sao oecuieg cece Ia sees) * Chemical used on contaminated counter tops + 89°F (31°C) for 30 minutes ‘+ Handwash agent opr catce 207 — DENA ex prneane- 2807 pone neon * Chlorine Chlorine is the active component of hypochlorite (bleach), which is used as a disinfectant. Disinfectants are antimicrobial agents that kill (germicide) or prevent the growth (microbiostatic) of pathogenic microorganisms. Disinfectants are not considered safe for use on living tissue (as opposed to antiseptics which are) and are applied only to inanimate objects (counter tops, light handles, headrests, etc). Phenol was the original disinfectant used in hospitals, but is rarely used as a disinfectant today because itis too caustic. Formaldehyde (37% solution in water -» Formalin) denatures protein and nucleic acids. Note: Concentration and contact time are critical factors that determine the ‘effectiveness of an antimicrobial agent against a particular microorganism. Any or all of the three major portions of microbial cells can be affected: the cell membrane, “cytoplasmic contents (particularly enzymes), and nuclear material. + 250°F (121°C) for 15-20 minutes "These conditions will yield 45 Ibs. pressure of steam per square inch. Moist heat destroys bacteria by denaturation of the high protein-containing bacteria, The autoclave provides sterilization when used at 250°F for 15-20 minutes because it applies the heat under pressure, which greatly speeds up the denaturation process when ‘compared with boiling water. Usually only ten minutes is required to destroy all of the bacteria, but the increased time is allowed for penetration when the instruments are wrapped in thick towels. ‘The effectiveness of autoclaving is best determined by culturing bacterial spores. ‘Spore testing of autoclave units is recommended weekY. ‘Note: Precleaning is the most important step in instrument sterilization. Debris acts as a barrier to the sterllant and sterlization process. Ultrasonic instrument cleaning is the safest and most efficacious method of preclezning. + Handwash agent ‘The term antiseptic is used for antimicrobial agents that are applied onto living tissues. Liquid antimicrobial preparations for handwashing, such as chlorhexidine gluconate, pparachlorametaxyienol, iodophors, and triclosan are examples. Chemical solutions, sprays, or wipes applied onto inanimate counters or other other environmental surfaces are termed disinfectants. ++ Cationic detergents Detergents are “surface-active” agents composed of a long-chain, lipid-soluble, hydrophobic portion and a polar hydrophilic group, which can be a cation, an anion, or a nonionic group. ‘These surfactants interact with the lipid in the cell membrane through their hydrophobic. ‘chain and with the surrounding water through their polar group and thus disrupt the ‘membrane. Quaternary ammonium compounds (e.g., benzalkonium chloride) are cationic detergents They are used as disinfectants and antiseptics. Gran-positve bacteria are the most susceptible to destruction. These compounds are not sporicidal, tuberculocidal, or vricidal and are inactivated by anionic detergents (soaps and the iron in hard water) ‘Anionic surface-acting substances include synthetic anionic detergents and soaps. These ‘substances alter the nature of interfaces to lower surface tension and increase cleaning. Their primary value appears to be their ability to remove microorganisms mechanically from the skin surface, Note: Nonionic chemicals do not possess any antimicrobial properties. Important: Disinfection destroys all the microorganisms in a certain environment except for spores. Therefore, objects that have been disinfected may stil transmit a pathoger ‘microorganism. Sterilization, however is the complete elimination of all organisms including spores. MICROBIOLOGY / PATHOLOGY MICROBIOLOGY / PATHOLOGY ‘Which type of pathogens provide the ultimate test for efficacy of sterilization? ‘The proper time and temperature for dry heat sterilization is: + Bacteria + 320°F (160°C) for 2 hours + Spore-forming + 250°F (121°C) for 20-30 minutes + Viruses + 450°F (232°C) for § minutes + Fungi + 89°F (31°C) for 30 minutes xp e208 2007 — BTA DECKS MICROBIOLOGY / PATHOLOGY Ic MICROBIOLOGY / PATHOLOGY Ic The killing or removal of all microorganisms, including bacterial spores, is called: Al of the following statements concerning ethylene oxide sterilization are true except: + Disinfection ‘I is used extensively in hospitals for the sterilization of heat-labile materials + Cleaning such as surgical instruments and plastics + Sterilization + It kills by alkylating both proteins and nucleic acids + Wiping + It is a fast process (20-50 minutes) depending on the material to sterilized + It is very toxie to humans and is also flammable + Spore-forming Because bacterial spores are resistant to boiling (100°C at sea level) they must be exposed to. higher temperature; this cannot be achieved unless the pressure is increased. For this purpose, an autociave chamber is used in which steam at a pressure of 15 IbJin, reaches fa temperature of 121°C and is held for 15-20 minutes. This kills even the highly heat- resistant spores of Clostridium botulinum, the cause of botulism, with a margin of safety. Saturated steam (aufociave) has proven to be the most practical, the most economical, {and the most currently effective sporicide. Its also the most efficient method for destruction of viral and fungal microorganisms. The steam autoclaves are made to operat inthe following ranges: + 1219 (250%) ata pressure of 15 pounds per square inch (ps) for 16-20 minutes. + 134°C (270°F) at a pressure of 30 psi for a minimum of 3 minutes (“flash cycle’). To positively destroy all living organisms, the minimum, required temperature is 121°C (250°F). Note: The autoclaving time will vary directly with the type of the load placed into the chamber, The 3-minute “flash cycle” is best indicated for unwrapped instruments. When instruments are wrapped, a longer sterlizing cycle is required to permit adequate Penetration of steam for proper disinfection. + Sterilization —> refers to the absence of all Iving forms Disinfection is the killing of many, but not all, microorganisms. It does not include the destruction of spores. Tho term disinfectant is reserved for chemicals applied to inanimate surfaces (lab fops, counter tops, headrests, ight handles, etc.). They are not ‘considered safe for use on living tissue. Antiseptics are chemical agents similar to disinfectants, but they may be applied safely to living tissue. Note: Soap only removes microorganisms. Note: Remember the doctrine of sterilization -> Do not disinfect what you sterilize! Remember: 1. The immersion of dental instruments in cold disinfectants will not destroy spores or the hepatitis viruses (they are resistant to physical and chemical agents). 2. Liquids are generally sterilized by filtration. The most commonly used fiter is ‘composed of nitrocellulose and has a pore size of 0.22 um. This size will retain all bacteria and spores. Fillers work by physically trapping particles larger than the pore size, + Itis a fast process (20-50 minutes) depending on the material to be sterilized "This is false; sterilization with ethylene oxide is a slow process (10-16 hours) depending on the material to be stelized Ethylene oxide gas has been widely used as @ sterilization agent, particularly for prepackaged, disposable plastic ware in hospitals. This gas is very toxic to humans and is also flammable, making its general use limited. Exposure of materials to ethylene ‘gas must be performed in special sealed chambers. Items must be cleaned and dried thoroughly before the sterilization process. Advantages of ethylene oxide sterilization: + Highly penetrative + Does not damage heat-labile materials (rubber, cotton, plastic, etc.) + Evaporates without leaving a residue ‘+ Works well for materials that cannot be exposed to moisture Important: Ethylene oxide functions as an alkylating agent by irreversibly inactivating cellular nucleic acids (DNA) and proteins. + 320°F (160°C) for 2 hours Note: 340°F (170°C) for 1 hour is also effective. lems which are usually sterlized by dry heat can be autoclaved. They should be removed immediately after cycle to diminish the possibility of corrosion of the instruments and dulling of sharp points or edges (carbon stee! instruments). Dry heat destroys microorganisms by causing coagulation of proteins. ‘Advantages of dry heat: * Effective and safe for sterilization of metal instruments, + Does not dull or corrode instruments Disadvantages of dry heat: * Long cycle + Poor penetration *+ Will ruin heat-sensitive materials Note: Instruments must be dry before both dry heat sterlization and ethylene oxide sterilization, Water will interfere with the sterilization process. MICROBIOLOGY / PATHOLOGY MICROBIOLOGY / PATHOLOGY Ie Al of the following are advantages of using alcohols (70% isopropyl and 70% ethyl alcohol) as surface disinfectants except: ‘The greatest occupational health care worker risk for bloodborne infection is: + Hepatitis ¢ vis + They are bactericidal + Human Immunodeficiency Vius + They are sporicdal + Hepatitis 8 vius + They are tuberculocdal + Tuberculosis + They are economical ream 207 — ern neous oor 05-2207 BTA. DEEKS MICROBIOLOGY / PATHOLOGY MICROBIOLOGY / PATHOLOGY Ic Antimicrobial chemical agents which destroy microorganisms when applied onto Which of the folowing is used as a handwash agent? inanimate surfaces such as counter tops or lights, are called: * Chlorhexidine gluconate = Antiseptics “*Triclosan. + Sterilants + Isopropyl alcohol + Disintectants “Both chlorhexidine gluconate and triclosan + None of the above “+ Allof the above oprenteste- nr — pan pecs cpm e207 — DENTAL cs + Hepatitis B virus ‘The basis of the current standard (formerly ‘universal’ infection control precautions was. first recommended by the CDC in 1987. The purpose was to protect health care workers. (HCW) from occupational transmission of all bloodborne infectious diseases during provision of patient care. While many health professionals focused on the Human Immunodeficiency Virus (HIV) as the major risk, accumulated evidence clearly demonstrated that the hepatitis B virus (HBV) was responsible for infection in 10-30% of ‘exposed, susceptible HCW. Concentrations of HBV in the blood of a chronic earrier can range between 1,000,000 to 100,000,000 virions per ml, in contrast to significantly lower viral loads demonstrated for bot HIV-infected persons and person with aids. Viral concentrations detected in hepatitis C virus (HCV) infected individuals range between those noted for HBV and HIV. Thus, we target the most infectious bloodbome pathogen with our infection control standards. Precautions that minimize potential HBV spread, also by inference, prevent cross-infection of less infectious microorganisms. Remember: Semen, serum, amniotic fluid and breast milk are fluids that can transmit HIV. Accumulating evidence suggests that HIV is not transmitted by casual household or ‘social contact. Important: Contact with saliva, tears, or sweat has never been shown to result in transmission of HIV. + Both chlorhexidine gluconate and triclosan Both chlorhexidine gluconate and triclosan have been shown to exhibit an antii- crobial effect when used as handwash agents in health care settings. In addition, to a broad antimicrobial activity, they have the property of substantivity, that is, a residual action on washed tissues for extended periods. Isopropyl alcohol is also used for hand hygiene procedures, but products containing {60-80% alcohol DO NOT use water, hence the term waterless hand hygiene, + Disinfectants, Sterilization is defined as the use of physical or chemical procedure to destroy all ‘microbial life, including bacterial endospores. Disinfection is less lethal than sterilization, and involves a chemical process of ‘microbial Inactivation which destroys virtually all pathogenic microorganisms on inanimate surfaces (ie., counter tops, light handles, bracket trays), but not necessarily all microbial forms (.¢., spores). ‘The use of antisoptics involves chemical agents applied onto living tissues (ie., handwashing) to both remove accumulated transient microorganisms, and tem- porarily lower the concentration of normal, resident flora. ‘They are sporicidal "This Is false; they are not sporicidal. ‘Alcohol is probably the most widely used antiseptic and is used to reduce the number Cf microorganisms on the skin surface in the area of a wound. Alcohol denatures proteins, extracts membrane lipids and acts as a dehydrating agent, all of which Contribute to its effectiveness as an antiseptic. Even some viruses (lipophilic viruses only) are inactivated by alcohol. The drawbacks of alcohol are that it evaporates too ‘quickly and has diminished activity against viruses in dried blood, saliva, and other secretions on surfaces (this fs due to the presence of tissue proteins and glycoproteins which render alcohol ineffective). For the above reason, alcohols are not regarded as effective surface cleansing agents (ie,, cleaning a dental operatory following patient treatment). Notes: 1. isopropyl alcohol (90%-95%) is the major form in use in hospitals. 12. Ethanol (70%) is widely used to clean the skin prior to immunization o venipuncture. 3. lodine is the most effective skin antiseptic used in medical practice. It acts as an} oxidizing agent and combines irreversibly with proteins. MICROBIOLOGY / PATHOLOGY An antibacterial solution which directly kills bacteria is said to be: MICROBIOLOGY / PATHOLOGY ‘Tho marker microorganism for Intermediate surface disinfection is: + Bacillus stearothermophilus + Bactericidal + Pseudomonas aeruginosa * Bactoriostatio. + Hepatitis B virus + Substantive “+ Mycobacterium tuberculosis MICROBIOLOGY / PATHOLOGY Ie The antigens most responsible for an immediate Type | reaction to natural rubber latex. are: MICROBIOLOGY / PATHOLOGY Cleaning surfaces prior to disinfection in clinical settings is required to * Destroy all pathogens + Proteins. * Inhibit pathogen growth + Accelerators + Reduce the concentration of pathogens + Com starch powders + Weaken the virulence of pathogens + Antioxidants + Proteins + Reduce the concentration of pathogens ‘The simplest way to approach environmental surface disinfection is to adhere to a basic premise of aseptic technique —> clean it first. All disinfectant products include specific, label instructions for cleaning prior to disinfection. COnly a few of the more than 250 proteins found in the sap from the rubber tree Hevea brasiliensis are responsible for causing the Type |, immediate, IgE-mediated reactions to natural rubber latex. These are water-soluble macromolecules that can leach out of latex gloves when a person perspires, or be detected on the surfaces of other products Cleaning is defined as the physical removal of debris. containing natural rubber latex (NRL). ‘Two effects result from efficient cleaning: 4. A reduction in the number of microorganisms present. 2. The removal of blood, tissue bioburden, and other debris which can interfere with disinfection. + Mycobacterium tuberculosis + Bactericidal The use of bactericidal chemicals is preferable to those which are “statie”. The latter do ‘not directly kill or inactivate microbes, but instead inhibit their metabolism and replica- tion. These affected organisms can remain viable but inactive for extended intervals. Application of "cidal” agents or processes are designed to ensure microbial inactivation. Antimicrobial activity against Mycobacterium tuberculosis is recognized as a significant benchmark criterion for disinfectant effectiveness. While tuberculosis is not transmitted via inanimate environmental surfaces, the morphology and structure of the tubercle bacilli make them relatively resistant to penetration by a number of low-level disinfectant chemicals. Important: Bactericidal agents work best during the log phase of bacterial growth. Intermediate level agents, such as phenols, iodophors, hypochiorite, and certain preparations containing alcohols plus other cleaner disinfectant chemicals are able to penetrate the wax and lipid outer layers surrounding mycobacteria. MICROBIOLOGY / PATHOLOGY Ic MICROBIOLOGY / PATHOLOGY ‘The most efficient way to kill microbes is: The most common form of an adverse epithelial reaction noted for health-care profes- sionals is: + Cold sterilization + Initation dermatitis + Proper handwashing with sterilizing antiseptics + Type | immediate latex allergy + Heat sterilization + Type IV, delayed latex allergy ‘Immersion of contaminated items in chemical sterlants + Superficial fungal infections on the fingers cx enn — cea ocx roa e206 27 DEAL DEOKS MICROBIOLOGY / PATHOLOGY Ic MICROBIOLOGY / PATHOLOGY IG ‘The complete destruction of all forms of microbial life, including spores, BEST describes: One of your patients develops a Type | immediate allergic reaction to latex. When treat- ing them and wearing gloves from now on, you can: += Disinfection * Wear vinyl or nitrile gloves + Sterilization * Wear hypoallergenic latex gloves + Pasteurization + Get an exemption and not wear gloves + Sanitization + Refuse to treat them conytten0e- 7 — pas. neoKe ori e208 2097 TAL DECKS + Sterilization + Sterilization > the destruction or removal of all forms of life, wth particular refer fence to microbial organisms: the limiting requirement is destruction of heat resistant spores. “+ Disinfection -> the use of chemical agents to accomplish the destruction of patho- ‘genic microorganisms, but not necessarily all pathogen or resistant spores, on inan- imate surfaces. ‘+ Pasteurization —> the treatment of dary foods, such as mik, for shor intervals with heat, to kill certain disease-causing microorganisms; the target of pasteurization is the destruction of Mycobacterium tuberculosis ‘+ Sanitization -> the treatment of water supplies to reduce microbial levels to safe public health levels. + Irritation dermatitis ‘Anumber of published reports have cited data suggesting that between 20-30% of health ‘care workers suffer from occasional or chronic dermatitis on their hands, The most ‘common manifestation of the condition is imitation dermatitis, a non-specific immune reaction often caused by contact with a substance that physically or chemically dam- lages the skin. The condition can be aggravated by frequent handwashing, residual glove powder left on hands, and the harshness of repeated use of some antiseptic hand- ‘wash agents. Health care workers located in colder climates may also experience chap- ping during the winter months. + Wear vinyl or nitrile gloves ‘A wide variety of latex-alterative infection control items have appeared in the marketplace within the last 10 years. The most widely recognized are newer generations of vinyl or nitrile gloves which do not cross-react with latex allergens. Products with the designation “hypoallergenic are no longer to be labeled latex alternatives, as they ‘contain latex with a chemical coating over the latex. Note: Studies over recent years showed that not all latex-llergic persons were able to use hypoallergenic gloves, since many Type | allergic individuals sill developed allargic ‘manifestations vinen using these gloves. + Heat sterilization ‘The use of heat has long been recognized as the most efficient, reliable, biologically monitorable method of sterilization. During @ routine cycle using an autoclave, unsaturated chemical vapor sterlizer, or dry heat unit, cell death is accomplished via hheat inactivation of critical enzymes and other proteins within microbial cell. ‘The recommendation stating that all reusable items that come in contact witha patient's blood, saliva, or mucous membranes must be sterilized using heat is now routinoly ‘accepted and used in dental facilites. MICROBIOLOGY / PATHOLOGY Ic A particular kind of antimicrobial treatment, such as that for our drinking water which lowers the total microbial load to safe public health levels is called: “+ Antisepsis + Sanitization + Disinfection + Sterilization MICROBIOLOGY / PATHOLOGY Which of the following have been involved in the transmission of Hepatitis C2 + Accidental neediesticks + Blood transfusions + Drug addicts sharing contaminated syringes +All of the above Crm ene 207 — cena next MICROBIOLOGY / PATHOLOGY Ic Which ofthe following statements are true? “+ Human Immunodeficiency Virus (HIV) is the most infectious target of Standard (univer- sal) Blood Precautions ‘+ Hepatitis B Virus (HBV) is the most infectious target of Standard (universal) Blood Precautions ‘There are no Standard Precautions for health care MICROBIOLOGY / PATHOLOGY Te) Which phrase describing an aspect of infection control is net appropriate? + Exposure is not synonymous with infection *Do not disinfect when you can sterilize * Sterilization of all clinical instruments and inanimate environmental surfaces is mandatory + Known AIDS patients can be treated using Standard Bloodborne Precautions + Sanitization ‘+ Hepatitis B Virus (HBV) is the most infectious target of Standard (universal) Blood Precautions Antiseptic -> a chemical that can be administered safely to external body surfaces or mucous membranes to decrease microbial numbers. Antiseptics cannot be taken HBV is the most infectious bloodbome pathogen known, and infection control precau- intemally tions aimed at preventing this viral transmission, nave also been shown to be effective in preventing HIV and HCV cross-infection. Disinfectant —> a chemical agent used to destroy microorganisms on inanimate objects such as dishes, tables, and floors. Disinfectants are not safe for living tissues. Disinfection — the process of reducing the numbers of or inhibiting the growth of microorganisms, especially pathogens, to the point where they no longer pose a threat of disease. Sterilization -> the process of kiling (or removing) all microorganisms on an object or ina material (e.g., quid media) * Sterilization of all clinical instruments and inanimate environmental surfaces is mandatory +All of the above Hepatitis C virus (HCV) is transmitted primarily in infected blood (blood transfusions). Historically, parenteral drug abusers, persons recelving transfusions, organ recipients, and hemophiliacs receiving factor Vill or [X were shown to be at high risk of HCV infec- tion. More recently, persons receiving tattoos or undergoing body piercings have been infected via contaminated, unsteriized needies. Note: The most likely route for a dentist, to be infected with Hepatitis C from a patient is from a contaminated needlestick {as opposed to saliva or the inhalation of aerosols). Itis not possible, nor necessary, to sterilize all environmental surfaces which become ‘contaminated during patient care. In many instances, because of the relatively low risk ‘of microbial transmission, thorough cleaning of the surfaces is sufficient to break the ‘cycles of cross-contamination and cross-infection. Important: Hepatitis C is a significant risk factor for the development of hepatocellular ‘carcinoma, MICROBIOLOGY / PATHOLOGY Ic All of the following are advantages of using alcohols (70% isopropy! and 70% ethyl alcohol) as surface disinfectants except: MICROBIOLOGY / PATHOLOGY Ie All of the following statements concerning ethylene oxide sterilization are true except: * They are bactericidal + It is used extensively in hospitals for the sterilization of heat-labile materials such as surgical instruments and plastics += They are sporicidal + Itkills by alkylating both proteins and nucleic acids + They are tuberculocidal + Itis a fast process (20-50 minutes) depending on the material to sterilized = They are economical + It is very toxic to humans and is also flammable Capi one 207 — oar. DECKS MICROBIOLOGY / PATHOLOGY The greatest occupational health care worker risk for bloodborne infection is: MICROBIOLOGY / PATHOLOGY Ic Antimicrobial chemical agents which destroy microorganisms when applied onto inanimate surfaces such as counter tops or lights, are called: + Hepatitis C virus + Antiseptics + Human Immunodeficiency Virus * Sterilants: + Hepatitis B virus + Disinfectants Tuberculosis * None of the above Coren ozas- 207 — TA OCHS rere ozs. zo ea. ceoKs + Hepatitis B virus The basis of the current standard (formerly “universa’) infection control precautions was first recommended by the CDC in 1987. The purpose was to protect health care workers (HCW) from occupational transmission of all bloodborne infectious diseases. during provision of patient care. While many health professionals focused on the Human Immunodeficiency Virus (HIV) as the major risk, accumulated evidence clearly demonstrated that the hepatitis B virus (HBV) was responsible for infection in 10-30% of exposed, susceptible HCW. Concentrations of HBV in the blood of a chronic carrier can. range between 1,000,000 to 100,000,000 virions per mi, in contrast to significantly lower Viral loads demonstrated for bot HIV-infected persons and person with aids. Viral concentrations detected in hepatitis C virus (HCV) infected individuals range between those noted for HBV and HIV. Thus, we target the most infectious bloodbome pathogen with our infection control standards. Precautions that minimize potential HBV spread, also by inference, prevent cross-infection of less infectious microorganisms. Remember: Semen, serum, amniotic fuid and breast milk are fluids that can transmit HIV, Accumulating evidence suggests that HIV is not transmitted by casual household or social contact. Important: Contact with saliva, tears, or sweat has never been shown to result in transmission of HIV. + They are sporicidal they are not sporicidal ‘Alcohol is probably the most widely used antiseptic and is used to reduce the number ‘of microorganisms on the skin surface in the area of a wound. Alcohol denatures proteins, extracts membrane lipids and acts as 2 dehydrating agent, all of which ‘contribute to its effectiveness as an antiseptic. Even some viruses (ligophifc viruses ‘only) are inactivated by alcohol. The drawbacks of alcohol are that it evaporates too ‘quickly and has diminished activity against viruses in dried blood, saliva, and other secretions on surfaces (this is due to the presence of tissue proteins and glycoproteins which render alcoho! ineffective). This is f For the above reason, alcohols are not regarded as effective surface cleansing agents (e., cleaning @ dentel operatory following patient treatment), ‘Notes: 1. Isopropyl alcohol (90%-95%) is the major form in use in hospitals. [2. Ethanol (70%) is widely used to clean the skin prior to immunization or venipuncture, [3. lodine is the most effective skin antiseptic used in medical practice. It acts as an| oxidizing agent and combines irreversibly with proteins. = Disinfectants: Sterilization is defined as the use of physical or chemical procedure to destroy all microbial life, including bacterial endospores. Disinfection is less lethal than sterlization, and involves a chemical process of microbial inactivation which destroys virtually all pathogenic microorganisms on inanimate surfaces (Le., counter tops, light handles, bracket trays), but not necessarily all microbial forms (ie., spores). The use of antiseptics involves chemical agents applied onto living tissues (.e., handwashing) to both remove accumulated transient microorganisms, and tem- porary lower the concentration of normal, resident flora. + It Is a fast process (20-50 minutes) depending on the material to be sterilized “This is false; sterilization with ethylene oxide is a slow process (10-16 hours) depending on the material to be sterilized. Ethylene oxide gas has been widely used as a sterilization agent, particularly for prepackaged, disposable plastic ware in hospitals. This gas is very toxic to humans and. is also flammable, making its general use limited. Exposure of materials to ethylene gas must be performed in special sealed chambers. items must be cleaned and dried thoroughly before the sterilization process. Advantages of ethylene oxide sterilization: + Highly penetrative + Does not damage heat-labile materials (rubber, cotton, plastic, etc.) + Evaporates without leaving a residue + Works well for materials that cannot be exposed to moisture Important: Ethylene oxide functions as an alkylating agent by irreversibly inactivating collular nucleic acids (DNA) and proteins. MICROBIOLOGY / PATHOLOGY Ic ‘An infection caused by normally non-pathogenic microorganisms in @ host whose resistance has been decreased or compromised is known as a (an): ‘+ Nosocomial infection + Secondary infection + Opportunistic infection * Medical infection MICROBIOLOGY / PATHOLOGY Latex allergy risk factors include all ofthe folowing except: + Persons with multiple surgeries “Atopy “+ Rubber industry workers + Persons with an allergy to pollen * Persons with an allergy to bananas MICROBIOLOGY / PATHOLOGY Which of the following is used as a handwash agent? + Chlorhexidine gluconate + Triclosan. + Isopropyl alcohol + Both chlorhexidine gluconate and triclosan +All of the above MICROBIOLOGY / PATHOLOGY Itis recommended that face masks be changed: * Between patients *Daily + Twice per day Ic Ic + Persons with an allergy to pollen Published studies have demonstrated an Increased isk of developing an allergic reaction to either latex protain (ype for certain chemicals used inthe latex manufacturing process (ype IV) in cartain groups of people. Current information has not shown a cross-reaction between pollen allercies and water-soluble latex allecgens. Individuals who appear to be predisposed to ‘eadlly developing type | hypersensitivity reactions (1, who are atopic), however, can become sen- sitzed to latex allergens more readily than people wit few or no allergies. Note: Atopy is the genetic tendency to develop the classic allergic diseases — atopic dermatitis, allergic rhinlis (hay fever), and asthma. Atopy involves the capacity to produce IgE in response to ‘commen environmental proteins such as house dusimites, grass pollen, and food allergens. Remember: TH1 & TH2 calls are subsets of T-holper lymphocytes, involved in cell-mediated Immune responses. ‘TH! cells sacrete IL-1 and gamma interferon , which enhance cell-mediated responses and inhib- itboth TH2 subset col actviy and the humoral immune responses. THis inflammatory, produces 1L2, IFNgamma, TNFbeta, provides help to B-cels in IgG2a production, activates macrophages ‘and CTL and stimulates delayed-type hypersensitivites (Type IV hypersensitivity). ‘TH2 cells, tho other subset of T-helpor coll, are also involved in celkmediated immune respons- es. TH2 cell activity and secretions are thought to inhibit cell mediated responses and to enhance the humoral response, TH2 cells produce IL4 , IL5, 6, IL10 and IL13, which provide help to B-cells ‘and induce class switch to IgE and IgG1 , as well as Supporting eosinophils and mast cells. + Opportunistic infection The percentage of people living with a wide variety of immune compromised conditions continues to increase. Along with the clinical manifestations of those types of diseases there can be accompanying deficiencies in aspects of host immune defenses. The sever- ity of deficiency can range from mild to life threatening, and predispose the compromised person to infections by organisms which would not usually occur in other people with intact innate and specific immunity. Nosocomial infections are those that originate or occur in a hospital or hospital-ike set- ting. Nosocomial infections are primarily caused by opportunists, particularly by: + Enterococcus spp + Escherichia col + Pseudomonas spp. * Staphylococcus aureus ‘Secondary infection is an infection that occurs duting or after treatment of a primary infection. It may result from the treatment itself or from alterations in the immune system. For example, a yeast infection that occurs after antibiotic treatment of a bacterial infec tion is a secondary infection. The development of bacterial pneumonia following a viral Upper respiratory infection is another example. + Between patients Face masks should be changed at least with every patient and more often if heavy spatter is generated during treatment. The CDC recommends that masks must be ‘changed between patients or during patient treatment if the mask becomes wet or moist from within or outside. BFE (Bacterial Fitration Efficiency) measures the percent efficiency at which the face mask fiters bacteria passing through the mask. PFE (Particulate Filtration Efficiency) measures the percent efficiency at which the face mask fiters particulate matter passing through the mask, Fluid resistance is defined as the ability of a face mask’s material construction to mini- mize fluids from traveling through the material and potentially coming into contact with the user of the face mask. Fluid resistance helps reduce potential exposure to blood and body fluids caused from splashes, spray or spatter. * Both chlorhexidine gluconate and triclosan Both chlorhexidine gluconate and triclosan have been shown to exhibit an antimi crobial effect when used as handwash agents in health care settings. In addition, to a broad antimicrobial activity, they have the property of substantivity, that is, a residual action on washed tissues for extended periods. Isopropyl alcohol is also used for hand hygiene procedures, but products containing {60-80% alcoho! DO NOT use water, hence the term waterless hand hygiene. MICROBIOLOGY / PATHOLOGY Rapid heat transfer striization provides: + Avery fast cycle time + No dulling of cutting edges + Dry instruments ater cycle ‘+All of the above ‘co cate 207 — pent 08K MICROBIOLOGY / PATHOLOGY In healthcare what is the primary disease prevention measure? + Wipe-wipe * Spray-wipe-spray + Handwashing * Vaccines MICROBIOLOGY / PATHOLOGY Ic is recommended for all instruments that are used in the mouth. * Disinfection * High-level disinfection * Sterilization involving the use of heat * Both disinfection and sterilization involving the use of heat ‘zor o200- 200 — pea oecKs MICROBIOLOGY / PATHOLOGY Ic Personal protective equipment clinic jackets should be: + Short sleeve, high neck + Long sleeve, high neck + Long sleeve, turtleneck * Whatever your preference opens — DBA DEEKE + Handwashing Hands have long been recognized as one ofthe most important vehicles for microbial spread of is- tease. More than 100 years ago, Semmelweiss and Lisler suggested that the hands of medical pro- fessionals were sources of crose-infection with pathogenic bacteria and nosocomial infections. For health care workors handwashing s a primary disease prevention measure. The simple act of hand- washing can signficanly reduce the number of transient and normal microorganisms that coloniza hhost tissue, thus limiting the potential for spread of infection between health care providers. and patients. For routine handwashing, a vigorous rubbing together of all surfaces of lathered hands for atleast 10 ‘seconds, folowed by thorough rinsing under a stream of water is recommended. ‘The aim of handwashing is to remove microorganisms from the hands, preventing their potential \wansfer.Iis known that organisms survive and multiply on human hands, creating the opportunity to Infect others or the host. Handwashing reduces the number of transient organisms on the skin surface. ‘Although hands cannot be steriized, most transient organisms can be removed by 20 seconds of proper scrubbing with soap and water. Proper subbing would include vigorous motion withthe hands ‘rubbing together and fingers working in bstwoen tho finger wob space and inclusive ofthe dorsal and ventral surfaces of the hands. Microbes that reside in sweat ducts and hair folicies of the skin, how ‘ever, cannot be dislodged readily. Surveys show that one in five medical professionals carry poten- tilly pathogenic antbiotic resistant pathogens on his or her hands. Handwashing by medical profes- sionals occurs at only 30% ofthe ideel rate. Failure to wash one's hands before and aftr each patient ‘contact fs probably the most important contribute to the spread of infections. These microbes pose 2 threat to patients with reduced defenses, so scrubbing with an antiseptic prior to contact with these patients is usually recommended. +All of the above FDA-approved, forced air, dry heat convection ovens are appropriate for stelization of heat-stable instruments and other reusable items employed in patient care, They use @ higher temperature than other dry heat units, and there is controlled internal air flow within the chamber. In contrast to the traditional type of dry heat sterilizers, a rapid heat transfer unit can achieve sterilization of items in substantially shorter times, while stil offering the advantages of dry heat. ‘Advances in the design of the dry heat oven resulted in the development of the dry heat convection unit, which uses forced air at higher temperatures. This method of rapid heat transfer achieves sterilization in 12 minutes at 375°F (190°C) for wrapped items and in 6 minutes for unwrapped tems. = Long sleeve, high neck For optimal protection, clinic jackets or coats are required to be long sleeved and high necked. This requirement was installed to minimize the potential for exposed skin to contact, and therefore become contaminated with, a patient's blood, saliva, or other potentially infectious material + Sterilization involving the use of heat ‘The use of heat has long been recognized as the most efficient, reliable, biologically monitorable method of sterilization. During a routine cycle using an autoclave, unsaturated chemical vapor sterlizer, or dry heat unit, cell death is accomplished via heat inactivation of critical enzymes and other proteins within microbial cells. ‘The recommendation stating that all reusable items that come in contact with a patient's blood, saliva, or mucous membranes must be sterilized using heat is now routinely accepted and used in dental facilities. MICROBIOLOGY / PATHOLOGY Ic Because are the hardiest microbes, their destruction is required before the definition sterilization has been met. MICROBIOLOGY / PATHOLOGY Ic Sterilization is. the elimination of all microbial organisms including spore formers. Disinfection is the destruction of disease causing microorganisms not including spore formers. + Hepatitis B viruses “The first statement is true and the second statement is false + Bacterial endospores a ‘+ The first statement is false and the second statement is true + Mycobacteria + Both staternents are true + Vegetative bacteria + Both staternonts are false cont 0208-107 — oA DEES ‘ep 0s 207 — OETA DECKS MICROBIOLOGY / PATHOLOGY Oral Cav Which of the following has been shown to be the most effective antimicrobial agent for reducing plaque and gingivitis long-term? MICROBIOLOGY / PATHOLOGY Oral Cav All of the following bacteria may be etiologically related to dental caries except: += Streptococcus mutans + Stannous fluoride + Actinomyces viscosus + Phenolic compounds A CHRIS jobacillus actinomycetemcomitans (AA) * Quaternary ammonium compounds += Streptococcus salivarius + Streptococcus sanguis + Actinomyces naeslut + Actinomyces israeli * Lactobacillus casei + Chlorhexidine Its effectiveness may be explained by the fact that it leaves the greatest residual concentration in the mouth after its use. It is rapidly absorbed onto the teeth and the pellicle; and it is slowly released. It is approved by the ADA as an antimicrobial and antigingivitis agent. Examples include Peridex and PerioGard. The most common fo offect of chlorhexidine is temporary, superficial staining of oral structures. Other topical antimicrobial agents: ‘+ Stannous fluoride —> antimicrobial action appears to be related to the stannous (tin) ion rather than to the fluoride ion. It is available in gel form (e.9., Stop, Gel Kam). The ADA accepts stannous fluoride for anticaries activity but not for anti- plaque or antigingivitis purposes. + Phenolic compounds -» approved by ADA as an antimicrobial and antigingivitis agent. Example is Listerine. = Quaternary ammonium compounds -> not as effective as others in reducing plaque or gingivitis, seems to be best at eliminating bad breath (halifosis). Examples include Scope and Cepacol. + Bacterial endospores Even when sterilizer gauges display correct values for intemal conditions and chemical tors or integrators indicate that appropriate chamber conditions have been reached to achieve sterilization, the use of calibrated biological indicators (Bi) remains the main guarantee of sterilization. These preparations contain bacterial spores which are more resistant to heat than vegetative bacteria, viruses, and other described infectious microbes. + Actinobacillus Actinomycetemcomitans (AA) “Streptococcus mutans is the primary etiologic agent initiating dental caries. The bacteria that initiate caries must have the ability to produce extracellular insoluble glucans. Dextrans and mutans are types of glucans (they are both polymers of glucose). These are produced by Streptococcus sanguis, mutans, salivarius, and Lactobacillus species. Levans (fructans) are polymers of fructose and are produced by Streptococcus salivarius, mutans, and sanguis as well as Lactobacillus casel and acidophilus. Remember: Dextrans, mutans, and levans are synthesized from dietary sucrose by caariogenic and plaque bacteria primarily as extracellular polysaccharides. Note: Actinomyces viscosus and naeslundil cause root-surface caries. Actinobacillus species are gram-negative coccobacillary rods. Actinobacillus actinomycetemcomitans (AA) is found as part of the normal flora in the upper respiratory tract. Itis a rare opportunistic pathogen, causing endocarditis on damaged heart valves and sepsis. Important: AA is most commonly implicated with the etiology of the localized aggressive (juvenile) periodontitis and periodontitis in juvenile diabetes. + Both statements are true * Sterilization -» the destruction or removal of all forms of life, with particular reference ‘to microbial organisms; the limiting requirement is destruction of heat resistant spores, “Disinfection — the use of chemical agents to accomplish the destruction of patho- genic microorganisms, but not necessarily all pathogen or resistant spores, on inan- mate surfaces. + Pasteurization > the treatment of dairy foods, such as milk, for short intervals with heat, to kill certain, disease-causing microorganisms; the target of pasteurization is the destruction of Mycobacterium tuberculosis. + Sanitization — the treatment of water supplies to reduce microbial levels to safe public health levels. MICROBIOLOGY / PATHOLOGY Oral Cav MICROBIOLOGY / PATHOLOGY Oral Cav The principal oral site for the growth of spirochetes, fusobacteria, and other gram- negative anaerobes is Which species has beon implicated in the dental caries process? + Staphylococcus * Salive + Bacteroides + Calculus + Escherichia coli * The gingival margin * Streptococcus * The gingival sulcus rtm 007 — BTA. DEKE zor 2008-20 — NAL 0ECKS MICROBIOLOGY / PATHOLOGY Oral Cav All of the following statements concerning bacterial plaque are true except: MICROBIOLOGY / PATHOLOGY Young plaque is dominated by: Oral Cav + Gram-positive cocci * Itis the key etiologic agent in the initiation of gingivitis and periodontal disease + Gram-positive rods + Itis an accumulation of a mixed bacterial community in a dextran matrix + Gram-negative rods + It forms on a cleaned tooth within minutes + Filaments + It is composed of solids (80%; 95% of which are bacteria) and water (20%) + There are two categories: supragingival and subgingival plaque * Different bacteria may be found in plaque (cocci, rods, and filaments) and their proportions change with time, diet, and location + Gram-positive cocel “Streptococcus sanguis is usually one of the first colonizers of plaque. Dental plaque is the key etiologic agent in the initiation of gingivitis and periodontal, disease. It Is an accumulation of a mixed bacterial community in a dextran matrix. Young plaque is composed of: + Gram-positive cocci (40-50%) ~» Streptococcus species + Gram-positive rods (10-40%) > Lactobacillus species, + Gram-negative rods (10-15%) —» Fusobacterium species + Filamonts (4% or less) -> Actinomyces and Veillonella species As plaque ages: + The composition changes -> 50% gram-positive and 30% gram-negative. + The number of cocel decreases and the number of filaments increases. + The number of aerobic bacteria decreases and the number of anaerobic bacteria increases due to reduced oxygen tension (decrease in available oxygen) due to its consumption by the early gram-positive colonizers. “= The gingival sulcus ‘The gingival sulcus is an area of stagnation and bacterial proliferation. Factors that contribute to this are 4) an increase in crevicular (gingival) fluid 2) desquamation of epithelial cells, and 3) bacterial acid products, ‘The normal healthy mouth consists mainly of obligate and facultative anaerobes, and acidogenic bacteria. itis essentially these same bacteria which are found in the normal healthy gingival sulcus that become opportunistic and influence the course of periodontal disease. Important: > Streptococci found in dental plaque: * S, Sanguis (produces hydrogen peroxide) * S. Mutans (itis aciduric and produces lactic acid) * 5. Salivarius (itis found consistently in saliva and on oral soft tissue) * S. Milis (produces hydrogen peroxide) predominantly ***Lactobacillus casel is also acidogenic (produces lactic acid). Remember: Streptococci are gram-positive coccl that are facultative anaerobes and are the most numerous group of bacteria in the oral cavity. + Its composed of solids (80%; 95% of which are bacteria) and water (20%) “This is false; it is composed of water (80%) and solids (20%; 95% of which are bacteria). ‘Stages in plaque formation; 1.Formation of the pellicle (acquired pelle) -> The pellicle is a surface coating of salivary origin, which Is primarily protein in nature, with some carbohydrate complexes. It is essentially structureless and bacteria-free. It will form on a clean tooth surface within a matter of minutes because of its salivary origin. It also will form on crowns, dentures, porcelain teeth, etc. 2.Bacterial colonization -» The bacteria are deposited upon the policle in a ‘somewhat orderly fashion. The first group to appear on the surface of the acquired pelice is streptococei (gram-positive coca). They colonize in tremendously large numbers. Following establishment of the coccal forms of bacteria, rod-shaped microorganisms (Bacteroides and Fusobacterium) attach to and colonize the tooth surfaces. As the plaque matures, the bacterial morphology shits to include filamentous types (Actinomyces). 3.Maturation stage + Saliva continues to provide agglutinating substances and ‘other proteins o the intercellular matrix, and bacterial intercellular adhesion results, The crystalline structure will increase and eventually wil calcity (calculus). * Streptococcus Specifically, 8. Mutans, S. Sanguis (which is the most frequently isolated Streptococcus in the oral cavity), and S. Salivarius. These bacteria produce dextran sucrase (aiso called glucosyitransferase), which catalyzes the formation of extracellular glucans from dietary sucrose. Glucan production contributes to the formation of dental plaque. This dental plaque holds the lactic acid which is produced by these Streptococci against the tooth. This acid dissolves the hydroxyapatite crystals which form the enamel ofthe tooth, creating caries. Note: The major cariogenic property of S. mutans appears to be its abilty to produce the enzyme glucosyltransferase. Streptococci have these essential properties necessary for caries formation * They adhere to tooth surface (colonize on tooth surface). + They produce lactic acid, which will dissolve the enamel of the tooth. * They produce a polymeric substance (from the metabolism of carbohydrates), which ‘causes the acid to remain in contact with the tooth. Remember: -» Prerequisites for the development of caries: 4. Cariogenic bacteria 2. Susceptible host 3. Supply of substrate for lactic acid production MICROBIOLOGY / PATHOLOGY Which of the following statements are true concerning supragingival and subgingival plaque except: Oral Cav + Subgingival plaque can be attached or loosely adherent (epithelium associated) + Supragingival plaque is attached or tooth associated + Subgingival plaque is dominated by gram-negative rods + Supragingival plaque is dominated by gram-positive cocci + Supragingival plaque has more anaerobes than subgingival plaque ‘eprom ez 207 — Daa peaks MICROBIOLOGY / PATHOLOGY ‘Which statement is true concerning Interferons? + They are antiviral antibodies * They are antibacterial substances + They act to prevent the replication of a range of viruses by inducing resistance + They are not considered to be a nonspecific resistance factor cep ezes tar — oma eens MICROBIOLOGY / PATHOLOGY Oral Cav Al of the following statements concering calculus are true except: + Itis calcified or mineralized bacterial plaque + Itforms on natural teeth, dentures, and other dental prostheses + The surface is very rough and is covered by a layer of bacterial plaque + Inorganic material makes up about 10-15% of the composition while organic ‘material and water make up about 70-90% of the composition ‘Its main role in periodontal disease is to serve as a collection site for more bacteria ‘Copyigh ©2006-2007 — DENTAL DECKS. MICROBIOLOGY / PATHOLOGY Oral Cav Which one of the following organisms is commonly implicated with the etiology of acute necrotizing ulcerative gingivitis? + Streptococcus sanguis = Actinomyces israelii + Prevotella intormedia * Streptococcus uberis: copra oas zon — pBaLceeKe + Supragingival plaque has more anaerobes than subgingival plaque ***This is false; subgingival plaque has more anaerobes than supragingival plaque Supragingival plaque -> saliva and diet alter bacterial composition * Is attached or tooth associated + Consists primarily of gram-positive facultative anaerobic cocci + Streptococcus sanguis, Actinomyces viscosus, and naeslundii predominate. As this plaque ages, vibrios, spirochetes, and gram-negative bacteria predominate. Subgingival plaque -> saliva and diet do not alter bacterial composition + Can be attached or loosely adherent (epithelium associated) + As pockets form, gram-negative anaerobic rods tend to prevail + Bacteria found includes Actinomyces species, Fusobacterium nucleatum, Trepon- ‘ema species (spirochetes), and Veillonelia Notes }. increased bacterial plaque -> increased gingival inflammation I2. Decreased bacterial plaque -> decreased gingival inflammation Important: The progression from a healthy gingival sulcus to gingivitis is associated with a shift towards gram-negative anaerobic rods. + They act to prevent the replication of a range of viruses by Inducing resistance Interferons are @ family of inducible glycoproteins produced by eukaryotic cells in response to viral infections, These proteins are elaborated by infected host cells that protect non-infected cells from viral infections. Interferons do not block the entry of the Virus into a cell, but rather prevent the roplication of viral pathogens within protected calls. Note: Interferons themselves are not antiviral antibodies. They have no direct effect on viruses, and their antiviral action is mediated by cells in which they induce an antviral slate. Interferon is considered a non-specific resistance factor (as are lysozyme, Complement, ec.) because interferon proteins do not exhibit specificity toward a partic- Ular pathogen, which means that interferon produced in response to one virus is also effective in preventing the replication of other viruses. + Provotella intermedia [ANUG is @ condition which presents rather pathognomonic (charactorstc of a singlo disease) clinical signs and symptoms. The two most important clinical signs are: 1. Interproximal necrosis and pseudomembrane formation on marginal tissues. 2. History of soreness (pai) and bleeding gums caused by eating and brushing Othor signs and symptoms include a fetor oris (offensive odor), a low-grade fever, lymphadenopathy, and malaise. Note: The dominant WBC noted in the inflammatory Infltrate of ANUG Is the neutrophil. ANUG occurs most often in adults between the ages of 18-30. Factors which seem to Predispose someone to ANUG include a history of gingivitis, tobacco smoking, gross neglect (poor oral hygiene), fatigue, and stress. ‘The following bacteria are associated with ANUG: + Intermediate-sized spirochetes + Prevotella intermedia = Fusobacterium species + Selenomonas species Note: According to the American Association of Periodontics (AAP), ANUG is now correctly referenced simply as "Necrotizing Ulcerative Gingivitis’ without the “acute” qualifier. The National Boards may or may not reflect this change. *Inorganic material makes up about 10-15% of the composition while organic ‘material and water make up about 70-90% of tho composition “This is false; inorganic material makes up about 70-90% of the composition while organic material and water make up the remainder of the composition. ‘Three phases of the formation of calculus: 4. Pellicle formation -> it begins to form within minutes. 2. Plaque maturation -» microorganisms grow together to form a cohesive plaque layer. 3. Mineralization of the plaque -»> both supra- and subgingivally. The average time for this whole process to take place is 12 days. Note: Calculus is mineralized plaque. It is formed by bathing the plaque in a highly concentrated solution of calcium and phosphorus (-e., saliva). Subgingival calculus is dark due to pigments from biood breakdown. Inorganic components of calculus (70-90%) —> calcium and phosphates with small ‘amounts of magnesium and carbonate (these are derived almost entirely from saliva). Hydroxyapatite and fluoride are also present. ‘Organic components of calculus -> microorganisms (same as plaque), desquamated epithelial cells, leukocytes, and mucin. MICROBIOLOGY / PATHOLOGY MICROBIOLOGY / PATHOLOGY Cytokines are: All ofthe following are formed via the cyclooxygenase pathway except: + Insoluble mediators that play and important rll in blood clotting + Prostaglandins '* Soluble mediators that play an important role in immunity + Prostacyclin + Soluble mediators that play an important role in genetic recombination + Leukotrienes * Insoluble mediators that play an important role in glomerular fitration + Thromboxanes ony 0200-81 —eTAL OBR cri o2tn-2087 — o6WTAL DECKS MICROBIOLOGY / PATHOLOGY MICROBIOLOGY / PATHOLOGY Sub Histamine release within the body causes all of the following except: {All ofthe following statements conceming serotonin are true except: + Increased capillary permeability + It ls widely considered to be a neurotransmitter + Increased gastric secretion + Its present in the brain + Bronchiolar constriction + It is believed to play a role in temperature regulation, in sensory perception, and in the onset of sleep + Arise in blood pressure + Itis synthesized from the amino acid arginine cant 20-707 — DAL DECKS erro oon 2097 — 92m. 0€eK * Arise in blood pressure Histamine is formed from histidine by decarboxylation and is released from the coarse cytoplasmic granules found within tissue mast cells and basophils. In the early stages of acute inflammation, histamine mediates the contraction of endothelial cells, increasing vascular permeability. Histamine is liberated by dogranulation triggered by the following stimuli 1. The binding of specific antigen to basophil and mast cell membrane-bound IgE. 2. The binding of anaphylatoxins (C3a and C5a) to specific cell-surface receptors. on basophils and mast cells Nott Histamine is responsible for the principal symptoms of anaphylaxis. Remember: 1. Serotonin is synthesized from the amino acid tryptophan by enteroendoctine cells in the gut and bronchi. Its actions are similar to histamine. 2. Bradykinin Is a vasoactive kinin that mediates vascular permeabilly, arteriolar dilation, and pain (pain from inflamed tissues is associated with the roloaso of bradykinin). It is @ potent vasodilator and is produced by the action of kallikrein (generated by activated Hageman factor, factor Xlla) on an alpha 2 globulin (kininogen). Note: It may be involved in blood pressure regulation. + Soluble mediators that play an important role in immunity ‘cytokines have certain properties in common. They are all small molecular weight peptides or alycopeptides. Many are produced by multiple coll types such as lymphocytes, monocytes / ‘macrophages, mast colls, eosinophils, even endothelial cells ning blood vessels. Each individ- Lal eytokine can have multiple functions depending upon the cell that produces it and the target cell) upon which it acts (called plelotropism). Also, several different cytokines can have the ‘same biologic function (called redundancy). Cytokines can exert their effect through the blood ‘stream on distant target cells (endocrine), on target cells adjacent to those that produce them (paracrine) or on the same cell that produces the cytokine (autocrine). Physiologically it appears that most cytokines exert their most important effects in a paracrine and / or autocrine fashion, “Thoir major functions appear to involve host defense or maintenance and repair ofthe blood ele- ments. Cytokines aro catogorized by their major specific function(s). There are four major categories of cytokines: 1. Interferons are so named because they interfere with virus replication, 2. The tumor necrosis factors (TNF) are so called because injecting them into animals caus- ‘88 a hemorrhagic necrosis oftheir tumors. 2. The largest group isthe interleukins, so named because thelr fundamental function appears to be communication between (inter) various populations of white blood cells (leukocytes- leukins). 4, The colony stimulating factors (CSF) aro so named because they support the growth and diferentaon of various elements of the bone marrow. ‘It Is synthesized from the amino acid arginine is synthesized from the amino acid tryptophan. Serotonin (5-hydroxytryptamine) is synthesized from the amino acid tryptophan by ‘enteroendocrine cells in the gut and bronchi. Serotonin is a powerful vasoconstrictor and stimulates platelet aggregation (blood clotting). By far the largest amount of ‘serotonin is found in cells of the intestinal mucosa. Smaller amounts of serotonin ‘occur in platelets and in the central nervous system. In the CNS it acts as a neurotransmitter. Serotonin acts as an inhibitor of pain pathways in the spinal cord, and its action in the brain is believed to help control the mood of the person, perhaps even to cause sleep. ‘Notes: 1. There are different types of serotonin that are called receptor subtypes, identified as "5-HT* followed by a number. The serotonin found in the gut consists main! of 5-HT3 and 5-HT4 subtypes, Most of the serotonin found in the brain is of the} 5-HT1 and 5-HT2 subtypes; antidepressants tend to work on these subtypes. 2, Serotonin Is secreted in tremendous quantities by carcinold tumors (tumors ‘composed of chromaffin tissue). 3. Lysergic acid diethylamide interferes with the action of serotonin in the brain. * Leukotrienes ‘The prostaglandins and a number of related substances (prostacyclin, thromboxanes, and leukotrienes) are chemical messengers. One or another of them is present in almost every body tissue. They act primarily as local messengers that ‘exert their effects in the tissues that synthesize them. Arachidonic acid (2 20-carbon unsaturated fatty acid) is the major compound from which prostaglandins, prostacyclin thromboxanes, and leukotrienes are derived. ‘Arachidonic acid is a part of phospholipids in the plasma membranes of cells. When a call is stimulated by a neurotransmitter or hormone, a plasma-membrane enzyme called phospholipase A is activated, and this enzyme splits arachidonic acid from the phospholipids. Different metabolic pathways utllize different enzymes that convert arachidonic acid into the different messengers: * To form prostaglandins, prostacyclin, or thromboxanes, the cyclooxygenase pathway utilizes the enzyme cyclooxygenase. snes, the lipoxygenase pathway utilizes the enzyme lipoxy- MICROBIOLOGY / PATHOLOGY Al ofthe following statements concerning fibrinolysin are true except: + Itis also called plasmin + Itis a proteolytic enzyme derived from plasminogen + Itis essential in blood clot dissolution (fibrinolysis) + Its a component of the body's nonspecific disease mechanism + Itis the most important fibrinolytic protease MICROBIOLOGY / PATHOLOGY Cells / Org Tho immunity to infectious agents that is provided by serum lysozyme is: ‘+ Acquired and T-cell mediated = Acquired and B-cell mediated + Innate and IgG mediated * Innate and nonspecific ‘ep e200 207 — pea. oecKs MICROBIOLOGY / PATHOLOGY Sub Which of the following is an enzyme formed in the kidney and released into the bloodstream where it has an important role in activating angiotensin? + Plasmin + Fibrinogen. MICROBIOLOGY / PATHOLOGY Sub Which of the following are respiratory enzymes capable of undergoing alternate reduction and oxidation? + Pyrimidine nucleotides + Cytochromes + Reductants = Purine nucleotides mig ono 2087 — BENTH. EOS “Innate and nonspecific Lysozyme is a hydrolytic enzyme that is capable of digesting bacterial cell walls containing peptidoglycan. The enzyme is present in serum, tears, and phagocytic cells, (monocytes, histiocytes, neutrophils, macrophages, etc.) protecting the host from invading ‘microorganisms nonspecifically. This enzymes hydrolyzes peptidoglycan (also called murein), which is the rigid component of the cell wall in most bacteria. This layer is biochemically unique and is not found in archaebacteria or any eukaryotic coll “There are two parts to the peptidoglycan molecule: 1. Peptide portion -> is composed of short, attached, cross-linked peptide chains con- taining unusual amino acids. 2, Glycan portion -» forms the backbone of the molecule, it is composed of alternately repeating units of the amino sugars N-acetylglucosamine and N-acetylmuramic acid. “This is the portion that is attacked by the enzyme lysozyme. Note: Penicilin inhibits the terminal step in peptidoglycan synthesis. Protoplasts are cells, ‘that have had their cell walls and capsules removed by enzymatic (lysozyme) or antibiotic (penicilin) treatment. + It is a component of the body's nonspecific disease mechanism “This is false; fibrinolysin is not a component of the body's nonspecific disease mechanism (as are lysozyme, complement, efc.). Other terms not to be confused with fbrinolysin: + Fibrinogen is a plasma protein that is essential for the coagulation of blood and is converted to fibrin by thrombin and ionized calcium. + Plasminogen is the inactive precursor to plasmin that is present in tissues, body fluids, circulating blood, and within clots. + Fibrin is a stringy, insoluble protein responsible for the somisolid character of a blood clot. it serves as a template for fibroblasts to repair tissue and walls off the area to infection. It is the product of the action of thrombin on fibrinogen in the clotting process. Fibrinolysis restores blood flow in the vessels occluded by a thrombus and facilitates healing after inflammation and injury. Remember: In the presence of thromboplastin and calcium ions, prothrombin is converted to thrombin, which in turn converts fibrinogen to fibrin. Fibrin threads then entrap blood cells, platelets, and plasma to form a blood clot. = Cytochromes Cytochromes are generally membrane-bound proteins that contain heme groups and ‘carry out electron transport. They are found in the mitochondrial inner membrane of ‘eukaryotes, in the chloroplasts of plants, in photosynthetic microorganisms, and in bacteria, ‘The heme group is a highly conjugated ring system (which means its electrons are very ‘mobile) surrounding an iron ion, which readily interconverts between the Fe2* (reduced) ‘and Fe (oxidized) states. Cylochromes are thus capable of performing oxidation and reduction. Because the cytochromes (as well as other complexes) are held within membranes in an organized way, the redox reactions are carried out in the proper ‘sequence for maximum efficiency. Note: Cytochromes are chemically related to hemogiobin, Examples include: + Cytochrome oxidase -» the terminal enzyme in the chain of events that constitutes, cellular oxygen consumption. Found in mitochondria. + Cytochrome P_ 450 -> important in the metabolism of many drugs. Found in liver microsomes (which are small parties typically consisting of fragmented ‘endoplasmic reticulum to which ribosomes are attached). + Cytochrome b -» a cytochrome of the respiratory chain, + Cytochrome bs > a cytochrome in the endoplasmic reticulum. = Renin Renin is a proteolytic enzyme produced by and stored in the juxtaglomerular ‘apparatus that surrounds each arteriole asit enters a glomerulus. Renin secretion by special- ized cells in the kidney is controled by a number of factors, including the activity of sympa- thetic nerves to the kidneys and the renal arterial blood pressure (J pressure = + renin secre- ion), Ronin acts on the procursor substance angiotensinogen, which is manufactured by the liver, ‘and is present in the blood. Renin converts angiotensinogen to angiotensin |. In tun, angiotensin | is converted to angiotensin Il by an angiotensin converting enzyme (ACE) asso- Ciated with the walls of blood capillaries, particularly in the lungs. Angiotensin ilhas several very important functions: + Constricts resistance vessels (via Angiotensin II receptors) thereby increasing systemic, vascular resistance and arterial pressure. ‘+ Acts upon the adrenal cortex to release aldosterone, which in turn acts upon the kidneys. to increase sodium and fluid retention. ‘+ Stimulates the release of vasopressin (antidiuretic hormone, ADH) from the posterior ptu- itary which acts upon the kidneys to increase fluid retention. ‘+ Stimulates thirst centers within the brain + Facillates norepinephrine release from sympathetic nerve endings and inhibits nore nephrine re-uptake by nerve endings, thereby enhancing sympathetic adrenergic function. *+ Stimulates cardiac hypertrophy and vascular hypertrophy. MICROBIOLOGY / PATHOLOGY MICROBIOLOGY / PATHOLOGY Autolysis: Cells / Org Which organelle contains hydrolytic enzymes necessary for intracellular digestion? Cells / Org “Refers to degradative reactions in cells caused by intracellular enzymes indigenous to the cell * Lysosome * Golgi apparatus + Refers to the cellular degradation by enzymes derived from sources extrinsic to + Microbody the cell + Phagosome +s the sum of intracellular degradative reactions occurring after the death of individual cells within a living organism +s the death of single cells within clusters of other cells, cores 20" — DTA OEE ‘at 208-207 — 8A DECKS MICROBIOLOGY / PATHOLOGY ‘Which of the following is classified as an antifungal agent? MICROBIOLOGY / PATHOLOGY Cells / Org ‘The enzyme catalase is contained in which organelle within a cell? + Bacitracin + Nucleus + Amphotericin-B * Microbodies + Polymyxin-B * Golgi apparatus + Neomycin * Mitochondrion + Amphotericin-B8 ‘This te an antiungal agent given intravenously or orally for the treatment of severe systomic fungal infections caused by fungi such as Candida species. Bacivacin, polymyxin- and neomycin are anibi- ties, Ankbiotics in goneral do not have antifungal proper. Amphotorin B and Nystatn are polyene Antibiotics wich impair ergosterol synthesis, Ergostera isthe major steel of fungal membranes. Nota: Systemic amphoterin B fs associated wth a high incidence of kidnoy toxicity Fin oe Vem] (Fa eshenmes s Sper cant Important: Antifungal crugs affect cell membrane permeabliy, causing leakage of cellar constituents, wih leads to the death ofthe atfected cols. + Lysosome ‘Allysosome is a membrane bag containing digestive enzymes. When a call needs to digest food, the lysosome membrane fuses with the membrane of a food vacuole and squirts the ‘enzymes inside. The digested food can then diffuse through the vacuole membrane and enter the cell to be used for energy or growth. The only thing that keeps the cell itself from being digested is the membrane surrounding the lysosome. Note: Lysosomes are formed when the Golgi complex packages up an especially large vesicle of digestive enzyme proteins. Important points to remember: 1, The mitochondria Is a double membranous structure that is responsible for cellular ‘metabolism -» "the powerhouse of the cell. 2, Ribosomes are the site of mRNA attachment and amino acid assembly — protein synthesis, 3, Inclusions are transitory, non-living metabolic byproducts found in the cytoplasm of the coll, They may appear as fat droplets, carbohydrate accumulations, or engulfed foreign matter 4, The golgi apparatus or complex is composed of membranous sacs and is involved in the production of large carbohycrate molecules and lysosomes. 5, The endoplasmic reticulum functions in transportation within the call 6, The nucleus controls synthetic activites and stores genetic information. phagosome is a vesicle that forms around a particle (bacterial or other) within the phagocyte that engulfed it. It then separates from the cell membrane and fuses with and receives the Contents of eytoplasmic granules (lysosomes). This coupling forms a phagolysosome in which digestion of the enguifed particle occurs. + Microbodios Microbodies are bounded by a single membrane and are compartments specialized for specific metabolic pathways. Note: Microbodies are similar in function to lysosomes but are smaller and isolate metabolic reactions that involve hydrogen peroxide (HO). Two important families of microbodies are: 4, Peroxisomes -> transfer hydrogen to oxygen, producing hydrogen peroxide, and are generally not found in plants. 2. Glyoxysomes — are common in the fat-storing tissues of the germinating seeds of plants. They contain enzymes that initiate the conversion of fats to sugar, a process: that makes the energy stored in the oils of the seed availabe. Catalases are enzymes that catalyze the decomposition of hydrogen peroxide into water and oxygen. Aerobic bacteria which possess this catalase are able to resist the effects of H,02, Remember: Anaerobic bacteria lack either superoxide dismutase or catalase or both. ‘The anaerobic bacteria that possess catalase are able to resist the effects of HjO>. Note: Superoxide dismutase catalyzes the destruction of O, free radicals. it protects ‘oxygen-metabolizing cells against harmful effects of superoxide free radicals. + Refers to degradative reactions in cells caused by intracellular enzymes indigenous to the cell "This usually occurs after the death of the call, and is part of the final phase of apoptosis (also known as programmed cell death). Heterolysis refers to cellular degradation by enzymes derived from sources extrinsic to the cell (0.g,, bacteria, etc.) Necrosis is tho sum of intracellular degradative reactions occurring after the death of individual cells within a living organism. Note: Autolysin is an antibody that in the presence of complement causes lysis of the cells. Autolytic enzymes produced by the organism thon degrade the cell's own. cell wall structures. The result is that, in the presence of cephalosporins or penicilins, growing bacterial cells are subject to lysis because without functional cell wall structures, the bacterial cell is not protected against osmotic shock. MICROBIOLOGY / PATHOLOGY MICROBIOLOGY / PATHOLOGY Ab ‘Which antibiotic is not only effective against most staphylococe!, aerobic and anaerobic streptococci, but is most effective in treating infections due to bacteroides species? The standard prophylactic regimen for a nonallergic patients is: + Adults: 2.0 grams Amoxicillin one hour before procedure + Penicilin VK + Adults: 2.0 grams Penicillin Vk one hour before procedure + Enthromycin * Children: 100 mg / kg Amoxicillin one hour before procedure *Tetracycline * Children: 100 mg / kg Penicillin Vk one hour before procedure + Cephalexin (Kefiex) = Clindamycin ‘eoraoaze 290 penaL peck ‘era 0-207 — B2NTA DECK MICROBIOLOGY / PATHOLOGY Ab ‘The only drug that a patient can take is clindamycin, so the patient must be instructed to notify the dental office if he / she MICROBIOLOGY / PATHOLOGY Ab Which of the following conditions may predispose a patient to candidiasis? + Hormonal disorder ‘+ Develops @ hearing problem * Coronary condition ‘+ Has 5 or more watery stools per day + Immune deficiency disorder + Has trouble sleeping * Chronic respiratory condition + Develops headaches xp 200-707 — Dex DECKS coi 0208-197 —DSATAL OES “Has 5 or more watery stools per day -> psoudomersbranous colts ocr ypeeeastiiy cereal ype Bacterial ypeeenty Bacterial peenaty JAmoniilivClay. acid exrbaonprin \ecpta ico anc wat yates ace ypenenvty leche hist wal yo [ace i poay lewamycn ——s Pseedomembranous ote fat npercuty let typerety Temeyeine Donpeyelne rumcin) ce Sica NinocysineAnacor Aesun) Jssconate None Steen \seacetas Jeeciricidat Nephrotosiity Getanyein list Nepcoate iprsocece pro) asics Nines, edache Iscescin ‘ral sess Bterla Nerve tiorphenies akin Sc ohosomer [Becton ivod dere + Adults: 2,0 grams Amoxicillin one hour before procedure + Amoxicillin = Adults: 2.0 g orally 1 hour prior to appointment ~ Children: 50 mg / kg orally 1 hour prior to appointment Alleray to amoxicillin -> use Clin¢amycin, Cephalexin, or Azithromycin as follows: + Clindamycin = Adults: 600 mg orally 1 hour prior to appointment Children: 20 mg / kg orally 1 hour prior to appointment + Cephalexin = Adults: 2.0 g orally 1 hour prior to appointment = Children: 50 mg / kg orally 1 hour prior to appointment + Azithromycin = Adults: 500 mg orally 1 hour prior to appointment — Children: 15 mg /kg orally 1 hour prior to appointment ‘**azithromycin can also be used to treat a periapical abscess in a patient who has a history of hypersensitivity to penicillin. Immune deficiency disorder Candidiasis is an infection, usually of the oral cavity or vagina, with a candida species, usually C. albicans, which causes an inflammatory, pruritic infection. Deficiencies in the immune system (either pathologic or therapeutic) allows opportunistic organisms (e.g., Candida albicans) to proliferate. It is common, especially in patients who have a deficiency in T-lymphocytes, or who are receiving chemotherapy, and in immunosu- pressed individuals (A/DS patients). This yeast-like fungi is a normal inhabitant of the oral cavity and vaginal tract, however, its growth is normally competitively inhibited by the indigenous bacteria of these areas. ‘Notes: 1. A vaginal candidiasis infection can produce a thick, white discharge. 2. Angular cheiltis (bilateral uicers atthe corner of the mouth) has been linked to C, Albicans. Remember: Nystatin and clotrimazole are the two antifungals that are used as “swish and swallow” to treat oral candida infections. Nystatin (Mycostatin) is taken as an oral suspension to be swished around the mouth and swallowed. Clotrimazole (Mycelox) is taken as a troche (lozenge) wich is slowly dissolved in the mouth and swallowed. They work by binding to sterols in the fungal cell membrane, increasing permeability and permitting the leakage of intracellular components. This leads to the death of the affect- ed fungal coll + Clindamycin Clindamycin binds to the 50S ribosomal subunit, blocking bacterial protein synthesis. Its use is restricted by its side effects such as severe diarrhea and pseudomembranous colitis. These side effects are caused by the overgrowth of the bacterium known as Clostridium difficile. Clindamycin is bacteriostatic and is active against most gram-positive and many anaerobic organisms, including the anaerobic gram-negative bacteria Bacteroides fragilis. In dontistry, clindamycin is an alternate antibiotic in the following situations: * When amoxiclin cannot be used for the standard regimen for prevention of bacter- ial endocarditis in patients undergoing dental procedures. + For treatment of common oral-facial infections caused by aerobic gram-positive ‘+ For prophylaxis for dental patients with total joint replacement. Ciindamycin can be given to patients allergic to penicilins since there is no cross allergenicity. MICROBIOLOGY / PATHOLOGY MICROBIOLOGY / PATHOLOGY ‘Which of the following antibiotics is considered a broad-spectrum antibiotic? ‘The penicillins act to: ‘= Penicilin Vk ‘+ Weaken the bacterial cell wall to the point that the cell dies when it ruptures * Cefaclor (Ceclor) + Inhibit protein synthesis and suppress bacterial growth + Penicillin G + Disrupt bacterial protein synthosis destroying the bacterial cell + Disrupt specific biochemical reactions destroying the cell ‘ortacozto- 20? — pea. DEKE cert 208: 2nr— DEAL DECKS MICROBIOLOGY / PATHOLOGY Which penicilin is prescribed primarily in the treatment of severe penicillinase- producing staphylococcal infections? Ab MICROBIOLOGY / PATHOLOGY Ab Which two antibiotics are usually prescribed in the treatment of rickettsial diseases? + Methicilin += Tetracycline + Ampicilin + Polymixin B + Penicilin Vk + Erythromycin. + Carbenicilin * Chloramphenicol + Methicitin Methicilinis not frequently used because of the incidence of interstitial nephritis and the avail- abilty of equally efficacious alternatives (nafciin and oxecil).Itis given IV in severe penicli- hnase-producing staphylococcal infections. Remember: Penicillinase is produced by certain bacteria (e.g., some strains of staphylococcl) that render penicilin inactive. t degrades the beta lactam ring structure of penicilin, Structural modification of penicilin G (for example methi- cin), can render the molecule resistant to penicilinases, but may also narrow the spectrum of ‘action, limiting the primary use of such antibiotics to the treatment of infections caused by ‘Staphylococcus species. Other penicilinase-resistant penicillins include cloxacillin, dicloxacilin, nafcilin, oxacilin, ‘amoxicilin / c'avulonate potassium (Augmentin), ampicilin / sulbactin (Unasyn), piperacillin, I tazobactam(Zosyn), and ticarcilin /clavulonate potassium (Timentin). Notes: Meticiin-osstant Staph, aureus (MRSA) Ts a group of resistant Staph. bacteria thal tan bo lestreterng.Thaeo bectora are resitant tal the ponslinaserseant pent] Gdns end cephalospains, Such sane are usualy resistant ea wall fo eminoglyoostdes tauacycinan enjtvonyclo, and cindemyean In the past. venoomyan hab Best ved fagamat MRSA, However, nitoorgnisms resistant to vancomyeh have ben reported and is use has been crated _ Pee nt work only on growing cas that contain peidogycan in their cl wel. Tis i tiny poieln shows he restos bacterial acy against growing oran-postive bacterial {tay hee eich popltoghyean or muri layer tow col wa). Remember: Penicillin inhibits the terminal step in peptidoglycan synthesis. * Cefaclor (Cecior) Cefacior is a member of the cephalosporin family of antibiotics. The cephalosporins are penicilinske in actions against bacteria. They are bactericidal antibiotics and act like the penicilins in that they affect the bacterial cell wall during cell division such that closure does not occur. Bacteria eventually lyse, resulting in death of the cell Cephalosporins act against a wide range of gram-positive and gram-negative bacteria. Currently there are four generations of cephalosporins, Progression from first through the fourth generation is associated with a broadening of action against more gram-negative bacteria and a decreased activity against gram-positive bacteria. Important cephalosporins within each generation: . et => cephalexin (Keflex), cephradine (Velosef, cefadroxil (Duricef), cefazolin (Ancef) + Second -> cefaclor (Caclor), cefuroxime (Ceftin), cefoxitin (Mefoxin) + Third + cefixime (Suprax), cefoperazone (Cefobid) + Fourth -> cefepime (Maxipime) Note: Approximately 10% of individuals expressing allergy to the penicilin family of antibiotics will have cross allergenicity to the cephalosporins. ‘Tetracycline + Chloramphenicol ‘The rickettsia are small, gram-negative, aerobic, coccobacillary bacteria that are obligate Iniracolluiar parasites. Thay contain both RNA and DNA, vl viruses contain either RNA or DNA, Human rickettsial infection results from insect bites. The human target cell for al rickettsiae |s the endothelial cell of capiiries and other small blood vessels, Human rickettsial infections present as systemic symptoms of headache, myalgias, and fever, followed by a rash. They are traditionally divided into two groups as seen below. ST Spotted-Fever Group: Rocky Mountain spotted fever ‘Queensland tick fever ‘Boutonneuse fever, Kenya tick fever ‘Siberian tick fever Rickettsialpox. Typhus Group: Louse-bome typhus (epidemic nphus) Murine typhus (endemic typhus) Scrub typhus Qfever = Weaken the bacterial coll wall to the point that the cell dies when it ruptures Basic mechanisms of actions of antibiotics: Agents affecting bacterial cell wall: ~ Penicilin family = Cephalosporin family ‘Agents affecting bacterial DNA: = Quinolone family (Le, ciprofioxacin) ~ Metronidazole (Flagy)) Agents affecting bacterial protein synthe: = Tetracycline family Erythromycin family = Clindamycin = Chloramphenicol ‘Agents interfering with bacterial metabolic pathways: ~ Sulfonamides (sulfa drugs) MICROBIOLOGY / PATHOLOGY MICROBIOLOGY / PATHOLOGY Fungi Which genera of fungi is not responsible for causing dermatophytosis (tinea or ringworm)? Al of the following statements concerning fungi are true except: + They are prokaryotic and lack a cell wall * Trichophyton + There are two types: yeasts and molds + Microsporum + They can be dimorphic; that is, they have two morphologic forms + Histoplasma + They have a distinct nuclear membrane as part of the cellular structure + Epidermophyton + They have both asexual and sexual r yroduction capabilities, + Most fungi are obligate aerobes; some are facultative anaerobes; but none are obligate anaerobes MICROBIOLOGY / PATHOLOGY A fungal infection that may develop in people who have poorly controlled diabetes is called: Fungi MICROBIOLOGY / PATHOLOGY Fungi Alllof the following statements concerning fungal spores are true except: + Morphologic characteristics (0.9., the shape, color, and arrangement) of + Aspergillosis conidia are a useful aid for the identification of fungi + A conidium is an asexually formed fungal spore + Mucormycosis + Fungal spores are as resistant to heat as bacterial spores * Cryptococcosis + Fungal spores cause allergi in some people cprtacowo- 207 ppwa.oecKs em ez amar — cena eos + Mucormycosis - also known as phycomycosis or zygomycosis Mucormycosis is a relatively rare fungal infection caused by saprophytic mold (0.., Mucor, Rhizopus, and Absidia). These fungi are not dimorphic. They are morph- logically characterized by the lack of septa in their hypha. Patients with diabetic, ketoacidosis, burns, or leukemias aro particularly suscoptibe. It results in black, dead tissue in the nasal cavity and blocks the blood supply to the brain, leading to neurologic symptoms such as headaches and blindness. ther opportunistic fungi which normally fall to induce disease in most normal persons ‘but may do so in people with severely suppressed immune systems: * Cryptococcus -> Cryptococcus neoformans causes Cryptococcosis. C. neo formans is an oval, budding yeast and is not dimorphic. Cryptococcosis is more ‘common than other fungal infections. This infection is severe only in people with ‘underlying immune system disorders, such as AIDS. Cryptococcosis may spread, ‘especially to the meninges, where the resulting disease is cryptococcal meningitis. * Aspergillus -> Aspergillus species, espocially Aspergillus fumigatus, causes an aspergilloma ("Yungus ball’) in the lungs and Aspergillosis. Apergillus species exist only as molds and are not dimorphic. They cause pulmonary infections in people who have AIDS or have undergone organ transplantation. * Candida > Candida albicans, the most important species of Candida, causes thrush, vaginitis, and other diseases. C. albicans is an oval yeast with a single bud. ‘Overgrowth of C. albicans in those with impaired host defenses produces the candidiasis. + They are prokaryotic and lack a coll wall ‘This Is false; they are eukaryotic and have a complex coll wall. In addition, they are all gram-positive and grow in Sabouraud's medium and contain both DNA and RNA. ‘Two types of fungi: 1. Yeasts -> grow as single colls that reproduce by asexually budding. 2. Molds -> grow as long filaments (hyphae) and form a mat or mass which is referred to as mycelium. Dimorphism is a characteristic of some fungi, meaning that they form different structures at different temperatures. They exist as molds in the saprophytic, free- living state at ambient temperatures and as yeasts in host tissues at body temperature. These fungi include the major pathogens ~ Blastomyces, Histoplasma, Coccidioides, and Candida. Most fungl reproduce asexually by forming conidia (asexual spores) from the sides or fends of specialized structures called conidiophores. Different conidia help in the identification of fungi. Examples of asexual spores (conidia) include: arthrospores, chlamydospores, blastospores, and sporangiospores. ‘Some fungi reproduce sexually by mating and forming sexual spores. Examples of sexual spores include: zygospores, ascospores, and basidiospores. + Fungal spores are as resistant to heat as bacterial spores "This is false; unlike bacterial spores, most fungal spores are completely killed when heated ‘at 80°C for 30 minutes. Examples of asexual spores (conidia): + Arthrospores ~> formed by fragmentation of the ends of hyphae; ‘of Coccidioides immitis Chlamydospores -> are thick-walled and quite resistant; characteristic of Candida albicans = Blastospores > formad by budding, as In yeasts; multple bude are called pseudohyphae (characteristic of C. albicans also) + Sporangiospores -» formed within a sac on a stalk by molds such as Rhizopus and Mucor Je the mode of transmission Examples of sexual spores: + Zygospores -> single large spores with thick walls + Ascospores —> formed in sacs which are called an ascus. + Basidiospores -» formed extemally on the tip of a pedestal called basidium Important points to remember about fungi: 4. The response to infection with many fungi Is the formation of granulomas (as seen in coecilol- domycosis, histoplasmosis, blastomycoss, ot.) 2. The cell walls of most fungi contain chitin, glucans, and proteins, Note: Sterols ergostero)) are in the coll mombrane, 3. All fungi, except for those belonging to the class zygomycetes, are septated (which means the hyphae form transverse walls), + Histoplasma ‘These fungi (Trichophyton, Epidermophyton, and Microsporum) Infect only the skin, nails, and hair, Dermatophytosis (tinea or ringworm) is common among people who live in communities where a high standard of sanitation is difficult to maintain. The most effective anti«mycotic, (antifungal) agent is griseofulvin. Although griseofulvin is prescribed to treat infections of the skin, it cannot be applied as a cream and must be taken in pil form. nn ee ements ines capitis Microsporum spp, Trichophyton spp [Lesions, combs, tilet (eingworm ofthe scaly) eticles, headrests Tinea corporis [Epidermophytoa, Microsporum spp, Lesions, floors, shower | ringworm ofthe body) Trichophyton spp. stalls, clothing Tinea pedis [Epidermophyton, Tichophyton app. [Lesions floors, shoes (ringworm ofthe fet athlete's snd socks, shower oot) sas Tinea uaguium [rrichophyton spp Lesions (ringworm ofthe nails) Tines crus (rhgworm ofthe [Trichophyton spp, Epidermophyton | Lesions, athlete sup- eran Fock itch) ports MICROBIOLOGY / PATHOLOGY Fungi A\l of the following statements concerning coccidioidomycosis are true except: * It is caused by the inhalation of dust aerosols containing the Coccldioldes immitis arthrospores, which are highly infectious + Itis endemic in hot, dry regions of the S.W. United States and Central and South America * Is referred to as “valley fever” or "San Joaquin fever” ++ It.can be treated with penicillin * The primary infection or lesion is in the lung “+ Its by and large an inapparent and self-limiting infection in endemic areas ‘ete 200-207 — DENTAL OCHS MICROBIOLOGY / PATHOLOGY A single-celled parasite that causes amebiasis in humans is: * Giardia lamblia + Entamoeba histolytica ‘= Trichomonas vaginalis = Belantidium coli apy 0700-701 — DENTAL DEES MICROBIOLOGY / PATHOLOGY Fungi Aflatoxins are produced by: + Candida species + Coccidioides species + Aspergillus species + Histoplasma species crop 78-207 — Dena OCA MICROBIOLOGY / PATHOLOGY iP All of the following statements concerning malaria true except: + Itis an infection of white blood cells + Drugs taken for prevention are not 100% effective + Symptoms can begin a month after the infecting mosquito bite + Early symptoms are nonspecific and often are mistaken for those of influenza + Rapid diagnosis and early treatment are important, particularly for falciparum malaria, which is fatal in up to 20% of infected people ai 0282087 — BENTH. DECKS + Entamoeba histolytica Amebiasis is an infection of the large intestine caused by the nonflagellated protozoan Entamoeba histolytica. Entamoeba histolytica exists in two forms during its life oycle: the active parasite (trophozoite) and a dormant parasite (cyst). Acute intestinal amebiasis presents a dysentery (e., bloody, mucus-containing diarthea). Note: E. histolytica can also produce liver abscesses, Giardiasis is an infection of the small intestine caused by a flagellated protozoan Giardia lamblia. It also exists in two forms (trophozoite and cyst). Giardiasis is one of ‘the most common patasitic infections of the small intestine. itis more common in male homosexuals and in people who have traveled to developing countries. Trichomoniasis is a sexually transmitted disease of the vagina (in women) or urethra {in men) caused by a flagollated protozoan Trichomonas vaginalis. T. vaginalis only exists as a trophozoite. Trichomoniasis is one of the most common infections ‘worldwide. Symptoms are more common in women. Note: Eniamoeba and Trichomonas species are found in the oral cavity. They appear to be nonpathogenic when located here Note: Balantidium colt is a cliated protozoan that can infect the colon causing diarrhea with accompanying abdominal colic, nausea, and vomiting with bloody stools. + It can be treated with penicillin “This is false; Amphotericin B (an antifungal) is the drug of choice in treatment of Coccidioidomycosis. Fluconazole and itraconazole are also used to treat various fungal infections. Blastomycosis (also callod Gilchrists disoaso or North American blastomycosis) is caused by Blastomyces dermatitidis, which is a dimorphic fungus that exists as a mold in soll and ‘as a yeast in tissue, This fungus is endemic in North and Central America. It grows in moist soi that is rich in organic material, forming hyphae with small, pear shaped conidia. Inhalation of the conidia cause human infection. Histoplasmosis is caused by Histoplasma capsulatum, which also is a dimorphic fungus that exists as a mold in soil and as a yeast in tissue. Histoplasmosis is usually a result of inhaling contaminated air. In the U.S. itis endemic in central and eastem states, especialy in the Ohio and Mississippi River valleys. Infection is usually asymptomatic, but it can cause a granulomatous, tuberculosis-like Infection (primary form of disease). It is a {frequent cause of pulmonary nodules, The infection may spread throughout the body, and this disseminated form, though uncommon, is quite serious. }'. In infected tissues, yeast cells of Histoplasma capsulatum are found within macro. phages. 2. Histoplasmosis resembles TB, both clinically and pathologically 3. Histoplasma capsulatum produces chlamydospores. Histoplasmosis tomycosis are rarely acquired from another individual. + Itis an infection of white blood cells “This is false; it is an infection of red blood cells caused by the parasite Plasmodium. Malaria is spread by the bite of an infected female Anopheles mosquito, a transfusion ‘with contaminated blood, or an injection with a needle that was previously used by a person with the infection. Malaria is caused by four plasmodia: Plasmodium vivax, ‘ovale, falciparum, and malariae. P. vivax and P. falciparum are more common causes of malaria than P. ovale and P. malariae. Often the frst symptoms are a mid fever, headache, muscle aches, and chills (tu-ike symptoms). The enlarged spleen characteristic of malaria is due to congestion of sinusoids with RBC’s. In all types of malaria, the total WBC count is usually normal, however, there is hyperplasia of the lymphocytes and macrophages. Several antimalarial drugs (chloroquine, mefloquine, and primaquine) are available. = Aspergillus species Aflatoxins are coumarin derivatives produced by Aspergillus flavus that cause liver damage and tumors in animals and are suspected of causing hepatic carcinoma in humans. This toxin binds to DNA and prevents transcription of genetic information. Aflatoxins are ingested with spoiled grains and peanuts and are metabolized by the liver fo the epoxide, a potent carcinogen. Aspergillus species exist only as molds; they are not dimorphic. Aspergillosis is most commonly caused by one of the following species: A. fumigatus, A. niger, or A fiavus. This disease takes one of three forms: mycetoma (grows in lung cavities), invasive aspergillosis (begins in lungs, spreads to other organs), or allergic bronchopulmonary aspergillosis (allorgy to spores that produces asthmatic attacks). Nosocomial infections are infections acquired during hospitalization, unrelated to the patient's primary condition. These infections are offen caused by the following organisms: Candida albicans, Aspergilus, E. coli, Hepatitis viruses, Herpes Zoster Virus, Pseudomonas aeruginosa, Streptococci, and Staphylococci. Hospital-acquired bacterial infections are generally more resistant to standard antibiotic treatments due to prior selective pressure. MICROBIOLOGY / PATHOLOGY MICROBIOLOGY / PATHOLOGY Infections caused by certain nematodes results in: Humans may acquire Toxoplasma gondil by: + Marked neutrophilia + Swimming in contaminated water + Marked eosinophilia + Sustaining a dog bite + Marked basophilia + Ingestion of cysts in poorly cooked meat + All of the above * Airborne conidia comic 2200-207 — era oEoKS any om 107 — OANA DEORE MICROBIOLOGY / PATHOLOGY Syndr MICROBIOLOGY / PATHOLOGY Syndr Mallory-Weiss syndrome is bleeding from an arterial blood vessel in the upper gastro- intestinal tract, caused by a mucosal gastric tear at or near the point where the esoph- ‘agus and stomach join. It is most common in: The thymus gland and parathyroid glands are malformed and dysfunctional or missing altogether in: + Sjégren’s syndrome . ynant * DiGeorge syndrome ‘Women whorare preanst eg eee ‘= Men over age 40, especially alcoholics + Peutz-Jeghers syndrome +Pre-school-aged children + Post-menopausal women an one. 2087 vena cn Cri o208- 2087 — nena. neces + DiGeorge syndrome DiGeorge syndrome is a rare immunodeficiency disorder characterized by various congenital ‘abnormalities that develop because of defects that occur during eary fetal development. These defects occur in areas known as the 3rd and 4th pharyngeal pouches, which later develop into the thymus and parathyrold glands. Developmental abnormalities may also occur in the 4th branchial arch. Normally the thymus gland is located below the thyroid gland in the nack and front of the chest land is the primary gland of the lymphatic system, which is necessary for the normal functioning of the immune system, The parathyroid glands, located on the sides of the thyrold gland, are responsible for the maintenance of normal levels of calcium in the blood. The thymus’ and parathyroid glands are missing or underdeveloped in children with DiGoorge syndrome. The symptoms of this disorder vary greatly, depending upon the extent of the missing thymus and parathyroid tissue. The primary probiem caused by DiGeorge syndrome is the repeated ‘occurrence of various infections due to a diminished immune system. ‘Notes: 1. The absence of the thymus results in T-cell deficiency. These children have normal B| lymphocytes and form antibodies, but they have decreased or absent delayed-tyo hypersensitivity, [2. These children develop totany due to hypocalcemia (from the absence ofthe parathyroid), 5. Shy-drager syndrome (Mule system atrophy) is a rare degenerative condition. its symptoms are similar to those of Parkinson's disease in that pationts may move slowly, ‘be tremulous, and have a shulfing gait. ‘+ Marked eosinophilia. Nematodes are roundworms with a cylindrical body and a complete digestive tract. There are two categories of nematodes based on their primary location in the body: 4. Intestinal nematodes —> Enterobius (pinworm), Trichuris (whipworm), Ascaris (giant roundworm), and Necator and Ancylostoma (the two hookworms).. 2, Tissue nematodes > Wuchereria, Onchocerca, and Lao are called “filarial worms”. {An infestation isthe presence of parasites on the body (0.g., ticks, mites, and lie) orn the organs (e.g, nematodes or worms). Infections caused by certain nematodes cause marked eosinophilia (abnormally large numbers of eosinophils in the blood). Eosinophils do not ingest the parasites; rather, they attach to the surface of the parasites via IgE and secrete cytotoxic enzymes contained within their eosinophilic granules. ‘+ Men over age 40, especially alcoholics Mallory-Weiss syndrome is characterized by mild to massive bleeding from an arterial vessel due to a tear in the mucosa of the cardia or lower esophagus with vomiting of bright red blood. Mallory-Weiss syndrome causes approximately 5% of all upper gastrointestinal bleeding. These tears are usually caused by severe retching and ‘vomiting. It is most common after excessive intake of alcohol. The treatment varies with the severity of bleeding, GI bleeding usually stops spontaneously. The tear usually heals in about 10 days without special treatment. Surgery is rarely required. ‘Notes: }. Hiatal hernia is a protrusion of a portion of the stomach from its normal position in} the abdomen through the diaphragm. The cause is unknown; most people have| ‘minor symptoms. [2. Achalasia is a nerve related disorder of unknown cause that can interfere with two processes: the rhythmic waves of contraction that propel food down the esophagus | called peristalsis, and the opening of the lower esophageal sphincter. Difficulty| ‘swallowing both solids and liquids is the main symptom. [3. Acid reflux is a backflow of stomach contents upward into the esophagus. The most obvious symptom is heartburn. ‘Toxoplasmosis is an infection caused by the protozoan Toxoplasma gondil. Sexual reproduction by this parasite occurs only in the cells lining the intestine of cats. Eggs (cocytes) are shed in a cat's stool. People become infected by eating raw meat containing the dormant form (cysts) of the parasite. It may resemble a mild cold or infectious mononucleosis in adults. It is treated with sulfadiazine. Cryptosporidiosis is caused by the intestinal protozoan Cryptosporidium parvum. ‘The main symptom of cryptosporidiosis is watery diarrhea that is sometimes accompanied by abnormal cramps, nausea, and vomiting. It is most severe in immunocompromised patients (those with AIDS). In these patients the infection may be fatal. |. Pneumocystis carinii is a protozoan that causes pneumocystis pneumonia in| immunocompromised individuals (AIDS patients). This lung disease is often fatal Remember: Candidiasis, hairy loukoplakia, and cryptosporidium enterocolitis are also ‘opportunistic infections associated with Immunocompromised individuals (AIDS patients), Protozoa are a diverse group of eukaryotic, typically unicellular, nonphotosynthetio| microorganisms generally lacking a rigid cell wall. MICROBIOLOGY / PATHOLOGY tall, infertile male with small testes most likely has which of the following conditions? Syndr + Adrenogenital syndrome + Klinefelter's syndrome ‘Testicular feminization + Tumer’s syndrome MICROBIOLOGY / PATHOLOGY ‘A male infant is born at term, No congenital anomalies are noted at birth. A year later he now has failure to thrive and has been getting one bacterial pneumonia after another with, both Hemophilus influenzae and Streptococcus pneumoniae cultured from his sputum. Which of the following diseases is he most likely to have? Syndr + DiGeorge syndrome + Selective IgA deficiency + Epstein-Barr vius (EBV) infection + Acute loukomia + Xinked agammaglobulinemia (Bruton's agammaglobutinemia) MICROBIOLOGY / PATHOLOGY Chvostek’s sign and Trousseau's sign are reliable indicators of: Syndr + Bell's palsy + Botulism + Rickets =Tetany MICROBIOLOGY / PATHOLOGY Syndr The triad of findings found in SjSgren’s syndrome include al of the following except: + Associated connective tissue disorders (2.9, rheumatoid arthritis) + Xerostomia (dry mouth) + Nephrosclerosis + Keratoconjunctivits Sicca (dry eyes) Crh ob — cen. DCH + %linked agammaglobulinemia (Bruton’s agammaglobulinemia) ‘Agammaglobulinomia is a rare disorder characterized by the absence of the serum immunoglobulin IgG. Its associated with an increased susceptibility to infection. Three forms: 4. Transient — common in infancy before six weeks of age. Mature B-cells are temporarily unable to produce antibodies. 2. Congenital (Bruton's) > rare, sex-linked, and results in decreased production of antibodies. The treatment of this disorder involves the repeated administration of IgG to maintain adequate levels of antibody in the circulatory system. 3. Acquired —> usually occurs in association with a malignant disease like leukemia, myeloma, or lymphoma, In Bruton’s agammaglobulinemia all five immunoglobulins and circulating B-cells fare absent or deficient but T-cells are intact. It affects males almost exclusively and causes severe, recurrent infections during infancy. Patients with this disorder are deficient in antibodies and susceptible to repeated infections (mostly bacterial and fungal). This results from the failure of B-cells (B-lymphocytes) to mature and to differentiate into plasma cells which produce antibodies. Note: This failure to mature is caused by a mutation in the B-cell protein tyrosine kinase. += Klinefelter's syndrome Klinefelter’s syndrome is a chromosome abnormality that affects only men and causes hypog- onadism. A person's sex is determined by the X and Y chromosomes. Normally, men have an Xand a ¥ and women have two Xs, In Kiinefelter’s syndrome, a male has two X's and a ¥. The condition is common and affects 1 in 500 men. The infant appears normal at bith, but the defect usually becomes apparent n puberty when secondary sexual characteristics fail fo devel- 1p (or develop late). These individuals have small testes, enlarged breasts, a feminine distibu- tion of pubic hair, and frequently mild retardation. These boys tend to be tall, wth long iegs. This disorder is associated with advanced maternal and patemal age. ‘Tumer's syndrome is a birth defect caused by the absence or defect of an X chromosome, Which inhibits sexual development and usually causes Infertlty. The incidence is 1 out of 3,000 live births. Girls with Tumer's syndrome usually have short stature, webbing ofthe skin of the neck, absent or retarded development of secondary sexual characteristics at puberty, absence ‘of menstruation, coarctation (narrowing) of the aorta and abnormalities of the eyes and bones. ‘The condition is either diagnosed at birth because of the associated anomalies, or at puberty when there is absent or delayed menses and delayed development of normal secondary sexu: al characteristics. Examples of diseases that can be diagnosed by karyotyping: * Klinfelter’s syndrome —> XxY Trisomy 18 -> extra chromosome 18, + Down's syndrome -> extra chromosome 21 ‘+ Tumer’s syndrome -> XO + Nephrosclerosis **"The clinical presentation of diminished lacrimal and salivary gland secretion is often referred to as sicca complex. ‘Sjogren's syndrome is a disorder of unknown cause (some researchers say it is autoimmune) marked chiefly by chronic inflammation (caused by white blood cell Infitration) of the salivary glands and lacrimal glands. This usually progresses to fibrosis and atrophy of these glands. All three symptoms rarely occur in one patient. A definite diagnosis can be made only when at least two of the symptoms are present. It is less common than rheumatoid arthritis and more prevalent in women than in men, ‘Two forms of the disease are recognized: Primary Sjégrens syndrome Is sicca complex ‘without other autoimmune disorders present, and secondary Sjégrens syndrome occur when the patient presents with both sicca complex and other associated autoimmune disorders. Note: Occasionally, the lymphocytic infitration is massive and causes enlargement of the glands (this is called Mikuliez's syndrome). Important: The decrease in salivation may cause rampant caries reminiscent of radiation caries. This is a result of a shift toward a more acidogenic microflora. + Tetany Tetany is a clinical neurological syndrome characterized by muscle twitches, cramps, ‘and carpopedal spasm. When severe, laryngospasm and seizures develop. All of these signs and symptoms reflect irritability of the central and peripheral nervous systems. itis, usually associated with calcium deficiency (hypoparathyroidism), vitamin D deficiency or alkalosis. Acute hypocalcemia in the human being ordinarily causes no other significant effects besides tetany because tetany kills the patient before other effects can develop. ‘Notes: 1, Tetany normally will occur when the blood concentration of calcium reaches approx, imately 6 mg% (normal is about 10 mg%).Itis lethal at about 4 mg’, [2. Chvostek’s sign —> to check for Chvostek's sign, tap the facial nerve above the| mandibular angle, adjacent to the earlobe. A facial muscle spasm that causes the} patient's upper lip to twitch, confirms tetany. 3. Trousseau’s sign -> to check for Trousseau’s sign, apply a blood pressure cuff to} the patient's arm. A carpopedal spasm that causes thumb adduction and phalangeal} extension, confirms tetany. MICROBIOLOGY / PATHOLOGY ‘Match the nutrients on the left with the symptoms on the right that are caused by a deficiency ofthat nutrient. Note: Symptom may be used more than once. Syndr s =| ‘Sore tongue and cracks at edge of mouth (cheilosis) Pins-and-needles sensation, especially in feet Folic acid | ein Vitamin By Vitamin B, {Weakness and bleeding gums Vitamin Bi [Bone thinning Paiste | Anemia, pins-and-needles sensation Vitamin D brissue swelling (edema), usually legs Vitamin K Protein Tendency to bruise and bleed ‘epret o2t0-2007— Dea vECKS MICROBIOLOGY / PATHOLOGY Peutz-Jeghers syndrome: ‘+ Is a hereditary condition that affects only females ‘+s a hereditary condition that affects only males + Is a hereditary condition that affects both males and females equally MICROBIOLOGY / PATHOLOGY Which immunodeficiency disorder is sometimes called "bubble boy disease"? Syndr + Wiskott-Aidrich syndrome + Severe Combined Immunodeficiency Disease (SCID) + Ataxia-Telangiectasia = Hyper-IgE syndrome: MICROBIOLOGY / PATHOLOGY Tropical sprue is a (an): Syndr + Immunodeficiency syndrome + Malabsorption syndrome * Autoimmune syndrome * Myofacial pain-dysfunction syndrome + Is @ hereditary condition that affects both males and females equally Peutz-Jeghers syndrome is a hereditary condition in which many small lumps called juvenile polyps appear in a variety of sites in the Gl tract, most commonly in the small intestine (especially in the jejunum). itis characterized by melanin pigmentation of the ‘oral mucosa, especially of the lips and gums. The polyps in Peutz-Jeghers syndrome are not true neoplasms, but rather hamartomas. These polyps don't increase the tisk of cancer in the intestinal tract. However, people with this syndrome are at increased risk for cancer of the pancreas, breast, lung, ovary, and uterus. Notes: 11. Gardner's syndrome is a type of hereditary polyposis in which various types of noncancerous tumors occur elsewhere in the body as well as in the intestine. It carries a high risk of colon cancer. Turcots syndrome is characterized by polyps along with tumors of the central} nervous system. Pigmentation is also seen in Wilson's disease (also called hepatolenticula/ degeneration). This disease is an inherited disorder of copper metabolism. It is| characterized by cirrhosis of the liver, degeneration of the basal ganglia of the brain, and the deposition of green pigment in the periphery of the cornea. (ee symntomens TT Pins-and-needles sensation, especially in feet Sore tongue and eracks at edge of mouth (cheilasis) Vitamin Biz | Anemia, pins-and-needles sensation |VitaminC | Weakness, bleeding gums Vitamin — [Bone thinning Vitamin | Tendency to bruise and bleed Protein Tissue swelling (edema), usualy in legs Remember: Malabsorption can cause deficiencies ofall nutrients or of proteins, fats, vitamins, for minerals selectively. The symptoms vary depending on the specific deficiencies. The absorption of the fat-soluble vitamins (A,0,E,K) is affected to the greatest extent, Vitamin Bz ‘malabsorption occurs in pernicious anemia due tothe absence of intrinsic factor. This is caused by the destruction or malfunction of gastric parietal cells. The term “pemicious” is currently a ‘misnomer since the disease is no longer fatal due to advances in treatment with parenteral ‘administration of vitamin By». Note: Vitamin B,, malabsorption also occurs with Crohn's disease, + Malabsorption syndrome, Malabsorption syndromes are disorders that develop because nutients from food are not being ‘absorbed propery into the bloodstream from the small intestine. People with malabsorption usually loge weight. Stestorrhea, which is stool that is light-colored, soft, bulky, and fou-smelling, is the ‘main characteristic finding in most malabsorption syndromes. ‘Malabsorntion disorders include: ‘Tropical sprue —> the cause of this disease is unknown, but it may be related to an infectious ‘organism. The condition aflects residents of of visors to the tropics. Typical symptoms include steatorthea, dlarthea, weightloss, and a sore tongue trom vitamin 8 defcioncy Celiac disease affects people aferently. Some people develop symptoms as chidren, others as sults. One factor tnought to play a role in when and how cellac appears is whether ana how long person was. breastfed -» the longer they were breasted, tho later the symptoms of celiac disease ‘appear, and the more atypical the symptoms, Other factors Includo the age at which one began eating ‘foods containing gluten and how much gluten was eaten, Noto: It can be fatal in adults due to tho {evelopment of lymphoma in the intestine. Whipple's disease is a rare condition that causes malabsorption due to infection ofthe intestine. It is ‘most likely caused by infection with Tropheryma whippell The disorder pimalyaflects middle-aged white ‘men, and the onset of symptoms is usually slow. Tey include skin darkening, inlamed and paint joints, ‘and dlathea. Without veatment, may be fatal. Intestinal lymphanglectasia|s 2 dlsordor of children and young aduls in which the lymph vessel ‘The most dangerous type of congenital (inherited) immunodeficiency, severe combined Immunodeficiency disease (SCID), results trom a fallure of stem cells to differentiate property. Individuals with SCID have neither 8 nor T-ymphozytes and are Incapable of any immunological response. These children generally cie before age two. Note: SCID became more widely known In tho 1970s when the world leamed of David Veto, a boy with SCID who lived for 12 years in a plastic, ‘germ-‘ree bubble. ‘The Wiskott-Aldtich syndrome (also called immunodeficiency with eczema and thrombocytopenia) affects only boys and is characterized by defective B-cell and T-cell functions. Its clinical features includ thrombocytopenia with severe blooing, eczema, recurrent infection, and an increased risk of Iymphoid cancers. Ataxie-telanglectasia fs an inherited disorder that affects many tssues and systems in the body. Muttole symptoms may include telanglectasis (lition of capillaries), ataxic (uncoordinated) galt. proneness to infection, defective humoral and eallular immunity and inereased risk of malignancies. ‘The most cbvious symptoms of the disease ore mutpletelangiectasos that are easily visible in the white of the eye and skin areas such as the ear and nose, graying of the halt, and iegular pigmentation of the areas exposed to sunlight. In addition, there is decreesed coordination of ‘moverens (ataxia) in late childhood Hyperimmunoglobulin E syndrome (also known as Job syndrome) is an Immunodeficiency healing by fibrous adhesion, without suppuration or granulation tissue formation. This occurs when wound margins are nicely ‘apposed, such as in surgical repair ofa surface wound. With welhapproximated wounds, there is litle granulation tissue, and the inal scar is minimal. Healing by second intention (socondary adhesion, secondary union) -> connective tissue repair ‘occurs when the wound is large and exudative, with a large amount of necrotic tissue and suppu- ‘ation formed as part of the inflammatory process. The site ils in witha highly vascular, pinkish is- ‘sue known as granulation tissue, which contains Roroblasts, endothalal cals, newly formed col- agen and capilary buds. This produces large, kregular scars, Healing by third intention -» the slow fling of a wound cavity or uloer by granulations, with sub- ‘sequent cicatrization (he process of scar formation) Notes: 1. The tensile strength of a healing wound depends upon the formation of collagen fibers. 2. Glucocorticoids have been shown to have the greatest effect on granulation tissue, ‘Whether a wound heals by primary intention or secondary intention is determined by tho} nature of the wound, rather than by the healing process itself, + Teratogenesis is not dose dependent "This is fal 9; teratogenesis is dose dependent. Teratogenic drugs act in different ways; they can inhibit, interfere, or block, metabolic steps critical for normal morphogenesis. Many drugs or viruses affect specific tissues or organs, Teratogenic agents: ‘+ Physical agents -> radiation, hypoxia, excessive carbon dioxide, and mechanical trauma = Maternal infection -» (TORCH complex) Toxoplasmosis, Other agents, Rubella, Cytomegalovirus, and Herpes simplex ‘+ Hormones -» sex and corticosteroids + Vitamin deficiencies — riboflavin, niacin, folic acid, and vitamin E ‘+ Chemotherapy drugs -» used for treating malignancies ‘+ Antibiotics -> mitomycin, dactinomycin, and puromycin (used as chemotherapy agents) TTeratogenesis may be induced when at least two conditions are met. Firstly, the teratogen must get into contact with the developing fetus. Secondly, the time these are in contact must be during the phase where the organ systoms are in process of being formed. This critical phase is the first 3 months folowing conception, also known as the first trimester. + Allograft (also called allogeneic grat, homologous grat, or homoplastic grat) Xenograft —> tissue from another species used as a temporary graft in certain cases, as in treating a severely burned patient. Itis quickly rejected but provides a cover for the bur for the first few days. In the skin graft rejection, the major host response is a cell mediated immune response (a delayed type IV hypersensitivity reaction). ‘Autograft -> surgical transplantation of any tissue from one part of the body to another location in the same individual. Also called autogenic graft, autologous graft, autoplas- tic graft, or autotransplant. Note: This type of graft has the best chance of success. Isograft -> composed of tissues taken from an individual of the same species who Is genetically identical (e.g, identical twins). ‘Notes: |. The most feared consequence of graft therapy in a patient with an immunodef-| ciency is a graft versus host reaction. J2. When a graft is rejected the first time and is tried again from the same donor, it wil be rejected more rapidly than the first.

You might also like